Sie sind auf Seite 1von 64

NOVEMBER

MONTHLE
NOVEMBER 2018

©LegalEdge Tutorials Page 1 of 60


Replication or other unauthorized use of this material is prohibited by the copyright laws of India
Index

Contents Page
No.
From the Directors’ Desk 3
A Note from the LegalEdge Team 4
Victory Story
“Not performing well in a mock was itself a source of inspiration for me” – Prakhar Pipariya 5
Current Affairs Timeline
National News 6
International News 8
Banking and Economy 9
Appointment & Resignations 10
Obituaries 11
Awards and Recognitions 11
Sports News 13
Defence and Space 14
Science and Technology 15
Current Legal Knowledge
Legal Affairs 16
Legal News 16
LE Explains
Disaster Management Act and Institute in India 18
General Election in India Constitution 19
Potpourri
20 Legal One Liners 20
Maxims and Foreign Phrases 20
Key Differences Between Important Concepts 21
LE Prep Assist
Mentor View (General Knowledge) 23
Mentor View (Mathematics) 24
Prep Test – Logical Reasoning 25
Prep Test – English Language 31
Current Affairs Practice Sheet CAPS
Current Affairs Practice Sheet (CAPS) 36
Answer Key with Explanations
Answer Key with Explanations Prep Test 47
Answer Key with Explanations (CAPS) 52

Published by: LegalEdge Tutorials. 127, Second Floor, Zone-II, M.P. Nagar, Bhopal – 462011
Contact No: 9111555433
www.legaledge.in

©LegalEdge Tutorials Page 2 of 60


Replication or other unauthorized use of this material is prohibited by the copyright laws of India
From the Directors' Desk
As we enter December 2018 and into the last 6 months of preparation, there are a few things that should be given due
care:
1. For all those who have received the CLAT-AILET Prep-Checklist of LegalEdge, go through the micro details carefully.
That's one of the most crucial documents of the session. Try and complete everything that's pending by 15 Jan, 19. For
all those who haven't, jigado :) Take the practice exercises of MonthLEs sincerely. MonthLEs are not only limited to
Current Affairs but extend to other sections as well. If you still haven't solved the MonthLEs (either its Current Affairs
Practice Sheets or its Prep Tests or anything else), its high time we completed them this month itself.
2. Furthermore, read a lot. Get into a variety of topics surrounding History: India after Independence, International and
National Current Affairs, Science, Technology, Economics and Business, Sports, Arts and Culture, Environment,
Religion and Literature. Ensure, that you touch base with every topic mentioned above via regular newspaper
readings (general articles, editorials, special features etc) and magazines. The more you read the better you will do in
English Language, Critical Reasoning and the Legal Reasoning Section. Moreover, solve a lot of MCQ questions on
Grammar to improve accuracy. Make regular notes of your grammar-errors. Make sure that that you atleast
complete the High Frequency Word-list.
3. Rather than doing a list of questions on a topic (of Mathematics and Logical Reasoning), club 4-5 of them and do all of
them together. If you are short of practice material in the form of offline tests, solving Past Years and Practice Papers
of SBI PO Prelims and Mains (and similar national level tests) will be useful for Mathematics as well as Logical
Reasoning. Assuming that we have done a lot of Maths and Logical Reasoning in the last few months, we would
suggest to go for a maximum number of sectional tests (under a time limit of 20mins - 20Q for Maths and 30mins-40Q
for LR) in the coming months. Say 2-3 tests for each section on a weekly basis should be very good.
4. General Knowledge and Current Affairs: Carry on the good work that you have been doing via MonthLE and a
website (preferably affairscloud). Do not ignore Static GK. Don't overdo it as well. :)
5. Revise Mocks. Try and solve your previous Mocks under a specified time limit. You may give yourself 80% of the
standard time and redo all questions. This will be specially helpful in Legal Reasoning. Don't search for a lot of
questions on Legal Reasoning. Redoing the set of mock-questions will improve your Legal Reasoning skills. You
should make a note of Principles that you find difficult to understand. Ask your teacher to explain them to the same
and you may pen down their explanation in your own words.
6. Constantly work on your weaker areas. The more you improve your weaker topics, the better are your chances of
increasing your scores. While going through your MAC book (Mock Analysis Copy book of LegalEdge) ponder over
sections-topics-areas which will fetch you more marks. Focus on them. Make a list of 10 things that can get you 5 more
marks - focus on those and repeat the exercise.
7. Cheat-code to score more in mocks:
Before any mock:
i) Solve 3 Reading Comprehensions, 30Q on Legal Reasoning and a 20Q (sectional test) on Mathematics. Revise your
grammar-errors.
ii) Revise the previous 2 months' Current Affairs thoroughly.
iii) 3 Puzzles in addition to 40Q (a sectional test) on Logical Reasoning.
If you do the above mentioned things in the morning before any mock, this would add atleast 7 more marks to your
overall score :)
Baaki sab theek? Bas lage raho! :)

Harsh Gagrani Karan Mehta


(Director, LegalEdge) (Director, LegalEdge)

©LegalEdge Tutorials Page 3 of 60


Replication or other unauthorized use of this material is prohibited by the copyright laws of India
A Note from the LegalEdge Team
Common Student Problems and their Solutions
Problem Solution
Sir number nahi bad rahe! It’ll take time. Rome wasn’t built in a day, similarly, we can’t expect scores
to increase within a few days/weeks. It’s a long journey ahead. Don’t get
demoralized. They’ll increase slowly but surely.
Sir Parents scold karte hai ki score nahi bad To make Parents happy in short run – Use Unfair Means in the next Mock
raha! 
To make Parents happy in long run – Work hard for the next six months
and get a good college in CLAT.
Choose one of the above as per your needs 
Sir jinke scores pehle kam they unke bahut Think about WHY is this happening?
bad gaye hai but mere nahi badh rahe! Answer can be ANY or ALL of the following:
 They’re consistently working harder and devoting a greater
number of hours everyday.
 They’re making fewer silly mistakes in the Mocks.
 They have a better background in conceptual subjects like English
and Mathematics.

Now, work on improving all of the abovementioned points in a


steady and your score will increase ultra-soon.

Some Valuable Pieces of Advice from the LE Team


1. Your scores will NOT increase consistently in every Mock and expecting it to increase in every Mock is WRONG.
2. Don’t analyze your MAC book or show it to the faculties after every Mock. Show it after 3-5 Mocks. That’s a good
enough time to decide whether or not and how much you have improved.
3. General speaking, study as per the following schedule:

Five Days a Week Current Affairs, Mathematics and English


Four Days a Week Logical Reasoning and General Studies
Three Days a Week Legal Aptitude

4. It’s recommended, but not ABSOLUTELY important to study any subject daily.
5. Persevere (find out what does this word mean!). It’ll pay!

©LegalEdge Tutorials Page 4 of 60


Replication or other unauthorized use of this material is prohibited by the copyright laws of India
Victory Story
“Not performing well in a mock was itself a source of inspiration for me”: Prakhar Pipariya

3. How did you motivate yourself for studying when


you didn’t perform well in your mocks?
Not performing well in a mock was itself a source
of inspiration for me. It always encouraged me to
work harder. After every disappointing result I
would get more dedicated to increase my marks
without being depressed. The mock results that
were the worst were the ones that kept me going
the most. Also, constant encouragement from the
faculties that there was still time to improve was
very helpful.
Prakhar Pipariya
(NLSIU Bangalore) 4. What was your revision strategy while preparing
for CLAT?
1. What made you choose a career in law? I did not have much in my mind about revision. But
From somewhere around middle of my 10+2, I had LegalEdge had distributed the course preparation
been pondering over various career options that lay so well that as they were revising things I was able
in front of me. Then my father advised me to do to revise them side by side. So I had not to worry
CLAT because of the multitude of opportunities much about revision. Grand Master’s Box that was
law offers. It is not limited to some specific field but quite good for revision.
allows you to select from a variety of careers. For
example, it is favorable to Civil Services 5. How instrumental has LegalEdge been to you for
Preparation. Apart from that, law is quite your preparation for CLAT?
fascinating and unique in its approach towards Legal Edge has well experienced teachers. They not
things. So, I decided to take a year’s drop and only teach very efficiently, but also help each and
prepare for CLAT. every student to perform better in mocks and learn
effectively. I learnt a lot from them. They were
2. Were you scared of any subjects in the beginning? available for doubts after the class. The mocks and
What did you do to overcome that? booklets had quite eclectic questions. It helped me
In my initial stages of preparation, I was quite build a good command over all the subjects. The
afraid of the English subject because it was difficult mock interface had a good analyzing system that
for me and I had very weak grammar. The basic helped me identify my strength and weakness
rules of English starting from parts of speech were properly. By constant hard work, coupled with well
unclear to me. I then engaged in a lot of reading and defined set of targets set by LegalEdge, I was able
question -solving activities. Since I did not get the to improve myself in the span of one year.
concepts initially, asking doubts was one of my
priorities. Reading materials suggested by my 6. What is your message for future CLAT aspirants?
teachers at LegalEdge, apart from the booklets I want to say to future CLAT aspirants that
given by LegalEdge, really helped me to improve a LegalEdge if a one step destination if anyone wants
lot and gradually I was able to pick up the to clear CLAT examination. The booklets and
language. I was also scared of the vast General mocks provided are self sufficient and one has no
Knowledge that CLAT examination covers but need to look for some other sources. Unceasing
MonthLE had all the important facts and a nice set effort and dedication are must but should be
of questions that relived me of my concerns. These coupled with proper guidance is available at
MonthLE covered many questions of not only LegalEdge. I strongly recommend LegalEdge to all
CLAT but various other law examinations. future CLAT aspirants.

©LegalEdge Tutorials Page 5 of 60


Replication or other unauthorized use of this material is prohibited by the copyright laws of India
Current Affairs Timeline

NATIONAL NEWS thermal imaging, ultrasonography, hydrotherapy


Lucknow stadium renamed after Vajpayee and quarantine facilities.
The newly-built ‘Ekana International Stadium’ Hookah bars permanently banned in Punjab
has been renamed after former prime minister Atal Hookah bars have been permanently banned in
Bihari Vajpayee in Lucknow. The stadium will now Punjab as President Ram Nath Kovind has given
be known as ‘Bharat Ratna Atal Bihari Vajpayee his assent to the Cigarettes and Other Tobacco
International Cricket Stadium’. UP Governor Ram Products Bill, 2018 to check use of tobacco. Punjab
Naik has approved the government proposal to this is the third state in the country after Gujarat and
effect. Maharashtra where hookah bars or lounges were
National Press Day banned through law.
National Press Day is observed on 16 November ‘Global talent rankings’ released
every year. On this day, Press Council of India The annual ‘Global Talent’ rankings was released
(PCI) started functioning as a moral watchdog. The by IMD Business School in Switzerland on 20
PCI was first set up in the year 1966 by the November 2018. The rankings are based on three
Parliament on the recommendations of the ‘First factors: Investment and Development, Appeal, and
Press Commission’. Readiness. India got the 53rd rank. Switzerland has
Food Park in Aurangabad district inaugurated topped the global ranking for the fifth year in a row.
Union Minister for Food Processing Industries Within Asia, Singapore got the 13th rank.
Harsimrat Kaur Badal inaugurated the second Mountaineer Satyarup climbed Mt Giluwe
Mega Food Park in Maharashtra on 15 November Indian mountaineer Satyarup Siddhanta became
2018. The Food Park is located in Paithan Taluka of the first Indian to climb Mt Giluwe on 11
Aurangabad district, Maharashtra. The first Food November 2018. The summit of the mountain is at
Park was inaugurated on 1st March 2018 in Satara an elevation of 4,367 metres. He had successfully
district, Maharashtra. climbed the Mt Everest in June 2016. Mt Giluwe is
Adobe CEO in Business Person of the Year list the second highest mountain in Papua New
Shantanu Narayen, the Chief Executive Officer of Guinea. Mt Wilhelm is the highest mountain of
Adobe has been named by Fortune in its ‘2018 Papua New Guinea.
Business Person of the Year’ list. In the list,
Shantanu Narayen is at 12th position. The list has
been topped by the CEO of insurance company
‘Progressive Tricia Griffith’.

J&K Assembly dissolved by Governor


Jammu and Kashmir Governor Satyapal Malik
dissolved the State Assembly on 21 November
2018. The Governor cited extensive horse-trading
and possible exchange of money and the
Human ancestor fossil discovered in Kutch
impossibility of forming a stable government by
Scientists have discovered a fossilised upper jaw
parties with opposing political ideologies as
of an eleven million-year-old human ancestor in
reasons.
Kutch, Gujarat. The find significantly extends the
Renaming of two divisions in UP approved
southern range of ancient apes in the Indian
The Uttar Pradesh government approved the
Peninsula. Apes, or hominoids, are a group of
renaming of Faizabad and Allahabad divisions as
primates from Africa and Southeast Asia that
Ayodhya and Prayagraj respectively on 13
includes the gibbons and the great apes: chimps,
November 2018. Prayagraj division will comprise
orangutans, gorillas, and humans.
Prayagraj, Kaushambi, Fatehpur and Pratapgarh
India’s 1st elephant hospital
districts. Ayodhya division will include Ayodhya,
India’s first specialised hospital for elephants was
Ambedkarnagar, Sultanpur, Amethi and Barabanki
formally opened in Mathura. The hospital is
districts.
located close to the elephant conservation and care
Statue commemorating Indian soldiers unveiled
centre and is designed to treat injured and sick
A new statue commemorating the role of Indian
elephants. The unique medical centre offers
soldiers in World War I was unveiled in France.
wireless digital X-ray, laser treatment, dental X-ray,
The seven-foot bronze statue commemorates over

©LegalEdge Tutorials Page 6 of 60


Replication or other unauthorized use of this material is prohibited by the copyright laws of India
4,700 soldiers and labourers from British India who were attacked by web-borne threats in India.
lost their lives on the Western Front during World Kaspersky Lab products detected 48,093,743
War I. Another memorial was inaugurated by Vice Internet-borne malware incidents on the computers
President Venkaiah Naidu at Villers Guislain in of KSN participants in India.
France. 1st India-Nepal passenger train
India allowed to buy Iranian oil: US The first passenger train to run on ‘broad gauge’
India is one of the eight countries to receive between India and Nepal is likely to run from
temporary exemptions from U.S. sanctions on Iran December 2018. The train will run from Jayanagar
that came into effect on 5 November 2018. The other in Bihar to Kurtha in Dhanusa district in south-
countries that have been exempted are China, Italy, eastern Nepal. An immigration check-post is likely
Greece, Japan, South Korea, Taiwan and Turkey. to be established at Jayanagar station. No visa will
U.S. said that each country on the list had be required for Indian and Nepalese nationals
demonstrated ‘significant reductions’ of the crossing the border through this stretch.
purchase of Iranian crude over the past six months. New record created in Ayodhya
INS Viraat to become a floating museum A new world record was set in Ayodhya on 7
The Maharashtra Cabinet approved the conversion November 2018 with over three lakh ‘diya’ lit up
of decommissioned carrier INS Viraat into a simultaneously. A total of 3, 01,152 diyas were lit
floating museum on 2 November 2018. The carrier up simultaneously for five minutes. The record was
will be housed on a concrete foundation near created at the Deepotsav event on the banks of the
Nivati Rocks in Sindhudurg in Konkan region of Sarayu river. The target was to illuminate a total of
Maharashtra. The project will be implemented 3.35 lakh diya on both sides of the ghats at Ram Ki
through public-private-partnership (PPP). INS Paidi.
Viraat was decommissioned in March 2017.

AR Rahman launched his biography


AR Rahman released his biography, ‘Notes of a
India elected as a Member of the ITU Council Dream: The Authorized Biography of AR
India has been elected as a Member of the Rahman’, in Mumbai on 3 November 2018. The
International Telecommunications Union (ITU) book has been written by author Krishna Trilok. AR
Council for another 4-year term (2019-2022). The Rahman has won six National Film Awards, two
elections to the council were held during the Academy Awards, two Grammy Awards, a BAFTA
ongoing ITU Plenipotentiary Conference 2018 at Award, a Golden Globe Award, and fifteen
Dubai, UAE. The ITU is a specialised agency of the Filmfare Awards.
United Nations (UN) that is responsible for issues Ayurveda Day
that concern information and communication Ayurveda Day was observed on 5th November
technologies. 2018. Ministry of AYUSH observes Ayurveda Day
IFFI to commence in Goa from 20th-28th Nov every year on ‘Dhanawantari Jayanti’ (Dhanteras).
The 49th International Film Festival of India (IFFI) On this occasion, Ministry is organising a ‘National
2018 will be held in Goa from 20th to 28th Seminar on Entrepreneurship and Business
November 2018. Israel has been selected as the Development in Ayurveda’ in association with
‘Country of Focus’ for 49th IFFI 2018. Jharkhand NITI Aayog on 4th & 5th November 2018.
has been selected as the ‘State of Focus’ for 49th IFFI West Bengal to observe ‘Rosogolla Day’
2018. Mr Dan Wolman from Israel will be honoured The West Bengal government will observe
with the Lifetime Achievement Award at IFFI. ‘Rosogolla Day’ on November 14. The day will be
Malayalam film Olu will be the opening film. observed to commemorate the first anniversary of
India ranked 12th in web-borne threats the State’s famous sweet getting Geographical
According to a recent report by Kaspersky Lab, Indication (GI) tag as ‘Bengal’s Rosogolla’. On 14
India is ranked 12th worldwide when it comes to November 2017, West Bengal got the GI tag for
the dangers associated with surfing the web. In the ‘Bengal’s Rosogolla’. The GI tag is a sign that
third quarter (July- September), 32.8% of the users
©LegalEdge Tutorials Page 7 of 60
Replication or other unauthorized use of this material is prohibited by the copyright laws of India
identifies a product as originating from a particular Oxford Dictionary has declared the adjective ‘toxic’
place. as the Word of the Year 2018. The adjective toxic is
Festival to celebrate Urdu heritage & culture defined as ‘poisonous’ and first appeared in
On 10 November 2018, the Delhi government English in the mid-seventeenth century from the
announced ‘Jashn-e-Virasat-e-Urdu’, a festival to medieval Latin ‘toxicus’. The Oxford Word of the
celebrate Urdu culture and its heritage. The festival Year is a word or expression that is judged to have
aims to keep the Urdu language alive and ensure lasting potential as a term of cultural significance.
its growth through larger participation. The festival India Pavilion Inaugurated at Videocittà 2018
will witness a diverse range of artists and art forms India Pavilion was inaugurated at Videocittà 2018
practising Urdu including traditional forms like in Rome. India was partner country at Videocittà
Chaar Bait, Dastangoi and Kissagoi. 2018. Videocittà 2018 is an event focussing on
24th Kolkata International Film Festival began Virtual Reality, Video gaming, Animation, Film
The 24th Kolkata International Film Festival began Making, etc. India’s participation at Videocittà
in Kolkata on 10 November 2018. A centenary 2018, Rome was organized by Ministry of
tribute will be paid to legendary Swedish director- Information & Broadcasting.
producer Ingmar Bergman through the screening
of eight of his outstanding movies. The event is
considered to be the country’s second oldest film
fest.

INTERNATIONAL NEWS
UN chief appointed new envoy to Syria
United Nations Secretary-General António APEC summit held in Papua New Guinea
Guterres has appointed veteran Norwegian Asia-Pacific Economic Cooperation (APEC)
diplomat Geir O. Pedersen as next Special Envoy summit was held on 17 and 18 November 2018 in
for Syria. He succeeds Italian-Swedish envoy Papua New Guinea. This is the first time Papua
Staffan de Mistura. He is currently Norway’s New Guinea, the poorest of the 21 countries in
ambassador to China. At the UN, Mr Pedersen APEC, has hosted the summit. The capital of Papua
served as the Special Coordinator for Lebanon New Guinea is Port Moresby and its currency is
between 2007 and 2008. ‘Papua New Guinean kina’.
‘World Malaria Report 2018’ released by WHO
World Health Organisation (WHO) has released
‘World Malaria Report 2018’ on 19 November 2018.
According to the report, India and 10 other
countries in sub-Saharan Africa accounted for
around 70% of estimated malaria cases (151
million) and deaths (274,000) in 2017. India
registered a 24% decrease in malaria cases in 2017
as compared to 2016.
Google launched ‘Neighbourly’ app Int day to End Impunity for Crimes
Google has announced the national roll-out of a The ‘International Day to End Impunity for
new app from its ‘Next Billion Users’ team called Crimes against Journalists’ is observed on 2
‘Neighbourly’. The app helps people source local November every year. The day draws attention to
information from their neighbours. Google is the low global conviction rate for violent crimes
rolling out ‘Neighbourly’ starting with Bengaluru against journalists and media workers. It is
and Delhi, which topped the waitlist. More cities observed in commemoration of the assassination of
will be added every day, including Chennai, two French journalists in Mali on 2 November 2013.
Hyderabad and Pune. 2019 World Corporate Games in Qatar
International Day for Tolerance Qatar will host the 23rd edition of the World
The ‘International Day for Tolerance’ is annually Corporate Games in 2019. The games will provide
observed on 16 November. The United Nations has an opportunity for corporations around the world
launched a new global campaign named to showcase their competitive prowess through
‘TOGETHER’ to promote tolerance, respect and sports. The World Corportate Games aims to bring
dignity across the world. together, the local, regional and international
Oxford Word of the Year

©LegalEdge Tutorials Page 8 of 60


Replication or other unauthorized use of this material is prohibited by the copyright laws of India
business sporting community. The last edition was signed a law that restricts the sale and use of
held in Houston in the US in 2017. sunscreen and skincare products that contain a list
Award given to Aung San Suu Kyi revoked of ten different chemicals. The ban comes into force
Amnesty International revoked the ‘Ambassador in 2020.
of Conscience Award’ given to Myanmar’s leader World Diabetes Day
Aung San Suu Kyi on 13 November 2018. She World Diabetes Day is observed on 14 November
received the award in 2009 when she was living every year. The theme of World Diabetes Day 2018
under house arrest. The award was revoked is ‘The Family and Diabetes’.
because of her failure to speak out for the UNSC agreed to lift sanctions against Eritrea
Rohingya minority. She was also stripped of her The United Nations Security Council (UNSC) has
honourary Canadian citizenship in October 2018. agreed to lift sanctions against Eritrea after nine
years. An arms embargo, asset freeze, and travel
ban were imposed in 2009 amid claims that Eritrea
supported militancy in Somalia. The resolution was
drafted by the United Kingdom and backed by the
United States and its allies.
International Cherry Blossom Festival began
The ‘India International Cherry Blossom Festival
2018’ was inaugurated in Shillong on 14 November
2018. It is the festival that highlights the pink cherry
flowers blooming all over the city in early
World Tsunami Awareness Day
November. A similar festival is also held in Japan
The ‘World Tsunami Awareness Day’ is observed
but it is celebrated in the spring when the cherry
on 5 November. In 2018, World Tsunami
trees bloom.
Awareness Day will align with the International
Tajikistan launched a hydro-electric project
Day for Disaster Reduction and the ‘Sendai Seven
Tajikistan inaugurated a US$ 3.9 billion ‘Rogun
Campaign’.
UNWFP & Alibaba entered into a partnership hydro-electric project’. It has been built on the
The United Nations World Food Programme Vakhsh River in southern Tajikistan. The project
(WFP) and Alibaba Group have entered into a will enable Tajikistan to eliminate domestic energy
strategic partnership to support efforts to achieve shortages and export electricity to Afghanistan and
the Sustainable Development Goal (SDG). Pakistan. The first of six turbines in the Rogun
Alibaba will provide its technology and resources hydroelectric dam went online on 16 November
to support the digital transformation of WFP’s 2018.
operations. Under the partnership, a digital World Africa’s first ever high speed train unveiled
Hunger Map to monitor the status of global hunger French President Emmanuel Macron and
will be developed. Morocco’s King Mohammed VI have inaugurated
World Science Day for Peace and Development Morocco’s first high-speed rail line, the first ever
World Science Day for Peace and Development is such line in Africa. The king Mohammed VI named
observed on 10 November every year. The day the first line as ‘Al Boraq’. It will connect the
aims to ensure that citizens are kept informed of economic hubs of Tangier and Casablanca. The $2
developments in science. The theme for 2018 is billion project was launched in September 2011.
‘Science, a Human Right’.
Pneumonia & Diarrhea Progress Report released
The 2018 ‘Pneumonia and Diarrhea Progress
Report’ was released by the International Vaccine
Access Center (IVAC) on 9 November 2018. The
report said that India’s vaccination coverage to
prevent rotavirus infection was the lowest among
the 15 countries. It also pointed out that India lost
over 2.6 lakh children under five years from ECONOMY AND BANKING
pneumonia and diarrhoea in 2016. World Bank’s ‘Ease of Doing Business Report’
Sunscreens banned in Palau released
Palau is set to become the first country to impose a According to the 2018 report, India is placed at 77th
widespread ban on sunscreen in an effort to protect rank among 190 countries assessed by the World
its vulnerable coral reefs. The government has Bank. In 2017, India was placed at 100th rank. In the

©LegalEdge Tutorials Page 9 of 60


Replication or other unauthorized use of this material is prohibited by the copyright laws of India
‘Grant of Construction Permits’ indicator, India’s Appeals for the DC Circuit is considered next to the
ranking improved from 181 in 2017 to 52 in 2018. U.S. Supreme Court. Ms. Rao is currently the
India’s economic growth to slow down in 2019 administrator of the Office of Information and
A report titled ‘Global Macro Outlook 2019-20’ Regulatory Affairs (OIRA).
was released by Moody’s Investors Service on 8 Flipkart CEO Binny Bansal quits
November 2018. The report said that the Indian Flipkart co-founder, Binny Bansal resigned as the
economy will expand 7.4% in 2018, but the growth company’s Group CEO following an independent
will slow down to 7.3% in 2019. It also said that the investigation into allegations of serious personal
economy grew 7.9% in the first half (January-June) misconduct. Kalyan Krishnamurthy will continue
of 2018. to be the CEO of Flipkart. Ananth Narayanan will
Fitch retains rating for India at ‘BBB-minus’ continue as CEO of Flipkart’s fashion arms Myntra
Fitch Ratings retained India’s sovereign rating at and Jabong.
‘BBB-minus’. ‘BBB-minus’ is the lowest Ashok Kumar Gupta appointed as CCI
investment grade rating. Fitch forecasts India’s Chairperson
growth to rebound to 7.3% in FY19 and 7.5% in Ashok Kumar Gupta has been appointed as the
FY20. The Fitch Ratings added that a recent Chairperson of the Competition Commission of
analysis conducted by it found that India had the India (CCI). He would be replacing the acting
highest medium-term growth potential among the Chairperson of CCI Sudhir Mital. CCI is a statutory
largest emerging markets. body of the Government of India responsible for
First overseas blockchain payment executed enforcing the Competition Act, 2002.
HSBC has executed India’s first financial Hima Das appointed as Youth Ambassador
transaction using blockchain for a deal involving Hima Das has been appointed as UNICEF India’s
an export by Reliance Industries to Tricon Energy. Youth Ambassador. She will work towards
The blockchain platform integrated with Bolero’s supporting various initiatives by UNICEF. She won
electronic Bill of Lading (eBL) platform to issue and the gold medal in Women’s 4 x 400 m Relay event
manage an eBL. This allowed a digital transfer of at the 2018 Asian Games.
the title of goods from the seller to the buyer. Kim Jong Yang elected as Interpol President
Kim Jong Yang of South Korea has been elected
APPOINTMENTS AND Interpol President. Kim will serve as president for
the remainder of the current mandate, until 2020.
RESIGNATIONS The decision was taken at the 87th General
Tesla names new chair to replace Elon Musk Assembly of the Agency in Dubai. He was
Electric carmaker Tesla has appointed Robyn previously the Chief of police in South Korea’s
Denholm as the Chairman of the company’s board most populous province.
of directors, replacing Elon Musk. She will also be
leaving her role as CFO and Head of Strategy at
Telstra, Australia’s largest telecommunications
company. She has been on the Tesla board as an
independent director since 2014. Elon Musk will
remain the Chief Executive Officer (CEO) of Tesla.

Youngest-ever goodwill ambassador appointed


British actress Millie Bobby Brown was named the
newest United Nations Children’s Fund (UNICEF)
Goodwill Ambassador. She is the youngest person
ever to be appointed as the UNICEF’s Goodwill
Ambassador. As a Goodwil Ambassador, she will
Neomi Rao to replace Kavanaugh on DC Circuit use her global platform to raise awareness of
U.S. President Donald Trump has nominated children’s rights and issues affecting young people.
Indian-American Neomi Rao to replace Supreme Abhijit Bose named India Head
Court Justice Brett Kavanaugh on the DC Circuit WhatsApp named Abhijit Bose as its India Head.
Court of Appeals. The powerful U.S. Court of He will join WhatsApp in early 2019 and will build

©LegalEdge Tutorials Page 10 of 60


Replication or other unauthorized use of this material is prohibited by the copyright laws of India
WhatsApp’s first full country team outside of Spider-Man, Daredevil and the X-Men. He teamed
California. He was the co-founder of Ezetap, an up with artists like Jack Kirby and Steve Ditko to
electronic payments company. revitalise the Comic industry.

OBITUARIES
Union Minister Ananth Kumar passed away
Union Minister for Parliamentary Affairs, and
Chemicals and Fertilizers H.N. Ananth Kumar,
passed away in Bengaluru. He is a six-time
member of parliament from Bengaluru South Lok
Sabha seat and was representing it continuously
since 1996. He held civil aviation, urban AWARDS AND RECOGNITIONS
development and tourism portfolios in Atal Bihari Couple honoured with ‘Roy M Huffington
Vajpayee cabinet during 1999-2004. Award’
Brigadier Kuldip Singh passed away Marie and Vijay Goradia received the ‘Roy M
Kuldip Singh Chandpuri, known as the hero of Huffington Award’ for 2018 in the US. They were
the Battle of Longewala during the 1971 Indo-Pak awarded for making significant contributions to the
war, died on 17 November 2018. Brig Chandpuri areas of literacy, education and healthcare in India.
was a recipient of the Maha Vir Chakra, the The Huffington Award is the highest honour
country’s second-highest gallantry award, for his granted by the Asia Society Texas Centre (ASTC).
role at Longewala, an outpost in the Thar desert of Vijay Goradia is the founder and Chairman of
Rajasthan. Brig Chandpuri was commissioned into Vinmar International.
the 23rd Battalion of the Punjab Regiment in 1963. Lilima Minz received ‘Ekalabya Puraskar’
Woman Hockey player Lilima Minz was
honoured with the prestigious ‘Ekalabya Puraskar’.
She was part of the squad that represented the
country in the 2016 Summer Olympics. The
Ekalabya Puraskar has been instituted by
‘IMPaCT’, the social wing of the Indian Metals and
Ferro Alloys.
Anupam Kher got ‘Distinguished Fellow’ award
Anupam Kher was honoured as a ‘Distinguished
Ad-filmmaker Alyque Padamsee passed away
Fellow’ by IndiaGlobal – a leading think-tank that
Prominent ad filmmaker and actor Alyque
works on issues concerning non-resident Indians.
Padamsee passed away on 17 November 2018 in
The actor was given a fellowship on November 3,
Mumbai. As a film actor, he was known for his role
2018 at the 3rd IndiaGlobal Summit. The event was
in Richard Attenborough’s film Gandhi where he
held at the prestigious MIT Sloan School of
played Muhammad Ali Jinnah. He was known as
Management in Boston, and was facilitated by MIT
the Father of modern Indian advertising. He was
Sloan India Business Club.
conferred with Padmashri Award in 2000 and he
Ram Mohan Roy award for N. Ram
also received Sangeet Natak Akademi Tagore
Veteran journalist and The Hindu Publishing
Ratna in 2012.
Group Chairman N. Ram has been chosen for the
Musician Ustad Imrat Khan passed away
prestigious Raja Ram Mohan Roy Award 2018. The
Classical music stalwart Ustad Imrat Khan has
award is presented by the Press Council of India
died in the U.S. on 23 November 2018. He
(PCI) for his outstanding contribution to
dedicated his life to promote the sitar and the
journalism. Ruby Sarkar, the chief correspondent of
surbahar worldwide. Ustad Imrat Khan belonged
Deshbandhu, and Rajesh Parshuram Joshte of
to the illustrious Etawa gharana, or the Imdadkhani
Daily Pudhari, have been named joint winners for
Gharana. The gharana is one of the oldest in India
‘Rural Journalism’.
with a musical legacy of over 400 years and traces
Namrata Ahuja bagged IPI-India Award
its roots to Agra.
The prestigious 2018 IPI India Award for
Stan Lee passed away
excellence in journalism has been given to
The comic book writer Stan Lee passed away on 12
Namrata Ahuja of ‘The Week’ magazine. She
November 2018. He is the co-creator of iconic
received the award for her exclusive story on Naga
characters including Iron Man, the Fantastic Four,
underground camps. The International Press

©LegalEdge Tutorials Page 11 of 60


Replication or other unauthorized use of this material is prohibited by the copyright laws of India
Institute (IPI) is a forum of editors, publishers and The prize has been constituted by the ‘Infosys
senior executives of newspapers and magazines. Science Foundation’ (ISF). The annual award
Arunima got Honorary Doctorate in UK includes a pure gold medal, a citation and a prize
On 6 November 2018, Arunima Sinha has been purse worth US$ 100,000. Navakanta Bhat was
awarded an honorary doctorate by the University awarded in the field of Engineering and Computer
of Strathclyde in the United Kingdom for her Science. Kavita Singh - Humanities. Roop Mallik -
inspirational achievements. On 21 May 2013, Life Sciences. Nalini Anantharaman -
Arunima Sinha became the world’s first female Mathematical Sciences. SK Satheesh - Physical
amputee to climb the Mount Everest. In 2015, she Sciences. Sendhil Mullainathan - Social Sciences.
was presented with the Padma Shri, India’s fourth- Dr. Martha received lifetime achievement award
highest civilian award. Late Dr Martha Farrell has been honoured with the
‘Lifetime Achievement Award’ at the 6th Indian
Social Work Congress in New Delhi on 13
November 2018. Dr Martha is being recognised for
her lifelong work towards gender equality,
women’s empowerment and prevention of sexual
harassment at workplace. In 1991, she co-founded
the NGO named Creative Learning for Change.
Anand conferred with ‘Global Education Award’ ‘India-Singapore Hackathon’ winners awarded
Mathematician and Super 30 founder Anand Prime Minister Narendra Modi felicitated the six
Kumar was conferred with ‘Global Education winning teams of the first ‘India-Singapore
Award’ in Dubai on 8 November 2018. He was Hackathon’. ‘India-Singapore Hackathon’ is a
awarded for his ‘pioneering’ contributions to the platform for the youth of the two countries to
field of education and illuminating the lives of harness and showcase their innovations. The
many talented students. The award was presented winning teams from India were IIT Kharagpur,
by ‘Malabar Gold & Diamonds’, a flagship NIT Trichy, and MIT College of Engineering,
company of Malabar Group, a leading Indian Pune.
business conglomerate. Ustad Amjad Ali Khan given a lifetime award
France’s highest civilian honour conferred Sarod master, Ustad Amjad Ali Khan was
Jawahar Lal Sarin was conferred with France’s presented the ‘Sumitra Charat Ram Award for
highest civilian honour by French Ambassador to Lifetime Achievement’ on 17 November 2018. He
India Alexandre Ziegler on 9 November 2018. He was awarded for his immense contribution to the
was given France’s highest civilian honour ‘Knight enrichment and promotion of the Indian classical
of the Legion of Honour’. He was awarded for his music. The award was presented to him by former
outstanding contribution to enhancing Indo-French diplomat Lalit Mansingh. He was awarded the
cultural cooperation and promoting the French Padma Vibhushan in 2001.
language.
Swati won ‘Press Freedom Award for Courage’
Journalist Swati Chaturvedi has won the 2018
Press Freedom Award for Courage. She won the
award for her book, ‘I am a Troll: Inside The Secret
World of The BJP’s Digital Army’. The award
ceremony was organised by the UK chapter of
Paris-based Reporters Sans Frontieres (RSF) or
CSE won Indira Gandhi peace prize
Reporters Without Borders in London.
Ladakh restoration project wins UNESCO award ‘Centre for Science and Environment’ (CSE) has
‘The LAMO Center’ in Jammu and Kashmir’s won Indira Gandhi Prize for Peace, Disarmament
Ladakh region has won an UNESCO Asia-Pacific and Development 2018. The award is conferred by
Awards for Cultural Heritage Conservation. It was Indira Gandhi Trust every year on the late Indira
chosen for its systematic restoration of an Gandhi’s birth anniversary. The international jury
aristocratic house in Ladakh. It won in the category for the awards was headed by former President
of ‘Award of Distinction’ under UNESCO Asia- Pranab Mukherjee.
Pacific Awards for Cultural Heritage Conservation. ‘Tata Literature Awards 2018’ announced
Six professors won Infosys Prize 2018 Anuradha Roy and James Crabtree are among the
winners of the ‘Tata Literature Live Awards 2018’.
Harper Collins India was named publisher of the
©LegalEdge Tutorials Page 12 of 60
Replication or other unauthorized use of this material is prohibited by the copyright laws of India
year 2018. Veteran journalist and writer Mark Ju Reti 6-1 in the final. He became the first Indian
Tully won the Lifetime Achievement Award. to win an Asian Snooker Tour event. The
Renowned poet Jayanta Mahapatra was named the Government of India has awarded him Padma
poet laureate for 2018. Bhushan in 2018.
Amitabh Bachchan received Sayaji Ratna Award Karen Khachanov won Paris Masters title
Amitabh Bachchan was awarded the third Sayaji Russia’s tennis player Karen Khachanov won the
Ratna Award. The award was established in the Paris Masters title on 4 November 2018. He
memory of erstwhile Baroda ruler Sayajirao defeated Novak Djokovic in the finals. He also
Gaekwad III. The Baroda Management won the Kremlin Cup in Moscow in October 2018.
Association had instituted the award to mark the He is currently ranked at 18.
150th birth anniversary of the ruler in 2013. Infosys
co-founder N R Narayana Murthy and noted
industrialist Ratan Tata have been given the award
earlier.
Vimal Chandra won the Young Scientist award
IIT Roorkee Professor Vimal Chandra Srivastava
has won the ‘NASI-Scopus Young Scientist Award
2018’ in ‘Environmentally Sound Sustainable Rahul Dravid inducted in ICC Hall of Fame
Development’ category. He has been given the Rahul Dravid became the fifth Indian to be
award for his work on Industrial Wastewater inducted in the ‘ICC Hall of Fame’ on 1 November
Treatment, Clean Liquid Fuels, and Multi- 2018. Rahul Dravid, along with Ricky Ponting and
component Adsorption. The Award was instituted England women wicketkeeper Claire Taylor, were
by Elsevier in the year 2006. announced as the new inductees in the Hall of
Fame. Dravid has scored 23, 208 international runs
for India across formats.
Subhankar Dey won SaarLorLux Open
In Badminton, Subhankar Dey has won the
SaarLorLux Open at Saarbrucken in Germany on
5 November 2018. He defeated Rajiv Ouseph of
Britain in finals. This is Subhankar’s first title of
2018. In March 2018, he had finished runner-up in
Filmmaker Nandita Das to receive FIAPF Award
the Kabal International tournament at the Czech
Filmmaker Nandita Das will be honoured with the
Republic.
‘International Federation of Film Producers
Angad Vir Singh won gold medal in shooting
Associations’ ( FIAPF) Award 2018. The award
Angad Vir Singh Bajwa won the gold medal in the
will be given at the ’12th Asia Pacific Screen
men’s skeet final of the 8th Asian Shotgun
Awards’ on 29 November 2018. She will be
Championship on 6 November 2018. He became
presented the award in recognition of her
the first Indian skeet shooter to win a gold medal at
achievement in film in the Asia Pacific region. She
any continental or world level event. Di Jin of China
started her career as an actor in films like ‘1947
won the silver medal while Saeed Al Maktoum of
Earth, ‘Fire’ etc.
UAE won the bronze medal.
Forbes India Leadership Awards 2018 announced
Bajrang becomes number one in world
The ‘Forbes India Leadership Awards’ was held on
On 10 November 2018, Indian wrestler Bajrang
November 22 in Mumbai. The award honours the
Punia achieved the number one rank in the world
top executives of companies in India. Azim Premji,
in the 65kg category. He was placed atop the
Chairman of Wipro Limited, got the ‘Lifetime
ranking table with 96 points in the United World
Achievement Award 2018’. Vivek Chaand Sehgal,
Wrestling (UWW) list. Alejandro Enrique from
Chairman of Motherson Sumi Systems, got the
Cuba is at number two with 66 points. Russia’s
‘Entrepreneur for the Year 2018 award’.
Akhmed Chakaev is at number three and Takuto
Otoguro from Japan is at number four.

SPORTS
Pankaj Advani won Asian Snooker Tour title
Pankaj Advani won the Asian Snooker Tour on 31
October 2018 in Jinan, China. He defeated China’s

©LegalEdge Tutorials Page 13 of 60


Replication or other unauthorized use of this material is prohibited by the copyright laws of India
the third Asian in last four years to get this honour,
after Byeong Hun An (2015) and Wang Jeunghun
(2016).
Lakshya won bronze medal in badminton
India’s Lakshya Sen settled for a bronze medal in
the World Junior Badminton Championship on 17
November 2018. He was defeated by Kunlavut
Vitidsarn of Thailand in the men’s singles
ATP World Tour Awards 2018 announced semifinals. Saina Nehwal is the only Indian to have
Novak Djokovic won the ‘Comeback Player of the won a gold in the ‘World Junior Badminton
Year Award’. Djokovic’s coach Marian Vajda was Championship’ during the 2008 Pune edition.
also named ATP Coach of the Year. Rafael Nadal First batsman to score 11,000 runs in Ranji
won the ‘Stefan Edberg Sportsmanship Award’ for Veteran cricketer Wasim Jaffer became the first
his fair play, professionalism and integrity on and batsman to score 11,000 runs in Ranji Trophy.
off the tennis court. Roger Federer won the Fans’ Jaffer, who plays for Vidarbha, achieved the feat on
Favourite player Award. day two of the Ranji Trophy match against Baroda.
11th Asian Shooting Championship 2018 winners He has played 31 Tests and two One-Day
Divyansh Singh won the silver medal in 10m Air Internationals for India. He last played for India in
Rifle Junior Men’s event. Elavenil Valarivan won April 2008.
the bronze medal in 10m Air Rifle Junior Women’s
event. Elavenil & Hriday won the gold medal in
10m Air Rifle Junior mixed event. Saurabh
Chaudhary won the gold medal in the 10m Air
pistol junior men’s event. Manu Bhaker & Saurabh
won the gold medal in the 10m Air pistol mixed
team event
Lewis Hamilton won Brazil Grand Prix
Formula One champion Lewis Hamilton won the DEFENCE AND SPACE
Brazilian Grand Prix. The win was Hamilton’s 10th India-Japan military exercise begins
of the season and 72nd of his career. Hamilton’s Armies of India and Japan began their first-ever
win helped Mercedes secure the constructors’ joint military exercise ‘Dharma Guardian-2018’ at
championship. Verstappen finished second while a jungle warfare school in Mizoram. The focus of
Ferrari’s Kimi Raikkonen took third. the two-week-long exercise will be to enhance
Shuttler Kento Momota won China Open tactical skills against global terrorism and increase
Kento Momota won badminton’s Fuzhou China inter-operability between the two forces. The
Open. He defeated Chou Tien-chen of Taiwan in Indian contingent is represented by 6/1 Gorkha
the final match. This was his 7th title of the year. Rifles.
The Fuzhou China Open is a part of the HSBC China unveiled ‘most advanced’ stealth drone
world tour events, and is designated as a super 750 China has unveiled its new-generation stealth
level tournament. combat unmanned aerial vehicle (UAV), UAV –
Alexander Zverev won ATP Finals in London CH7. The CH7 makes China the second country,
Tennis player Alexander Zverev won the ATP following the US, to produce high-altitude long-
Finals in London. He defeated Novak Djokovic in endurance combat UAVs with advanced
the finals. Alexander Zverev became only the penetration capabilities. The CH7 can intercept
fourth player to beat Roger Federer and Novak radar electronic signals, and simultaneously detect,
Djokovic back-to-back in the semi-final and final of verify and monitor high-value targets.
the same tournament. He also became the youngest Study finds that 3 moons orbit the Earth
winner of the ATP title. A team of astronomers and physicists has
Shubhankar won ‘Rookie of the year’ honours confirmed that the dust clouds orbit the Earth.
Indian golfer Shubhankar Sharma became the These dust clouds move with the Earth and Moon
first Indian to emerge as the ‘Sir Henry Cotton at a stable 4,00,000 kilometres from the Earth. The
Rookie of the Year 2018’ on the European Tour. He dust clouds is also known as ‘Kordylewski’ clouds.
follows Jon Rahm (2017), Wang Jeunghun (2016), The first glimpse of the clouds was seen only in
Byeong Hun An (2015) and Brooks Koepka (2014) 1961 by Polish astronomer Kazimierz Kordylewski.
who have been the four Rookies before him. He is

©LegalEdge Tutorials Page 14 of 60


Replication or other unauthorized use of this material is prohibited by the copyright laws of India
Milky Way. The name of the galaxy is Antlia 2 or
Ant 2 and is known as a dwarf galaxy. The Galaxy
was discovered with the help of Gaia satellite.
Despite its size, the galaxy has very few stars. Ant
2 is 10,000 times fainter, meaning that it has about
one 10,000th the number of stars.
New artillery guns inducted in Army ‘Super-Earth’ discovered by scientists
The Indian Army inducted its first new pieces of An international team of scientists has discovered a
artillery on 9 November 2018. The ‘M777 howitzer’ cold ‘super-Earth’ exoplanet orbiting around the
and ‘K9 Vajra-T’ were formally inducted into red dwarf Barnard, the second closest star system
service at the Deolali field firing ranges in to Earth. The new planet has been named
‘Barnard’s star b’ (or GJ 699 b). It orbits its red star
Maharashtra. M777 howitzers are US-made
lightweight guns, the K9 Vajras are South Korea- every 233 days near the snow-line, a distance where
water freezes. Barnard’s star is six light-years away
made self-propelled artillery guns. This is the first
major induction since the Bofors were inducted in from earth.
the 1980s.
NASA’s Ralph to explore ‘Trojan Asteroids’
NASA’s Ralph and Lucy are all set to explore
Jupiter’s Trojan asteroids. Ralph is a space
instrument that has travelled as far as Pluto, while
Lucy is a mission payload. The Lucy spacecraft will
carry a near-twin of Ralph, called L’Ralph, which
will investigate Jupiter’s Trojan asteroids. The
mission will be launched in 2021 and would be the
very first space mission to study the Trojans. SCIENCE AND TECHNOLOGY
NASA spotted new iceberg in Antartica India’s first microprocessor developed
NASA has spotted an enormous new Antarctic Researchers at Indian Institute of Technology-
iceberg while on a mission to survey the region’s Madras (IITM) have designed and booted up
shifting sea ice and glaciers. NASA’s Operation India’s first microprocessor, ‘Shakti’. The Shakti
IceBridge flight spotted the giant berg which microprocessor can be used in low-power wireless
broke off from Pine Island Glacier in October systems and networking systems. The Shakti
2018. The US National Ice Centre estimates the family of processors was fabricated at Semi-
iceberg named B-46, is 66 square nautical miles (87 Conductor Laboratory (SCL), Indian Space
square miles) in size. Research Organizations (ISRO) in Chandigarh.
GROWTH-India telescope’s first observation Paleontologists discover new Sauropod species
The ‘GROWTH-India telescope’ at the Indian Paleontologists have discovered the remains of a
Astronomical Observatory located in Hanle, dinosaur that lived 110 million years ago in
Ladakh has made its first science observation Argentina. The remains came from three separate
which is a follow-up study of a nova explosion. dinosaurs from the herbivorous group of
Novae are explosive events involving violent sauropods, the best known of which are the
eruptions on the surface of white dwarf stars, Diplodocus and Brontosaurus. This new species
leading to temporary increase in brightness of the has been named Lavocatisaurus agrioensis.
star. This recurrent nova has been named ‘M31N-
2008’.
Military exercise ‘Vajra Prahar’ commenced
The joint military exercise ‘Vajra Prahar’ between
India and the United States commenced in Jaipur
on 19 November 2018. The aim is to enhance the
interoperability of the two armed forces. The
armies will train on different aspects including Largest brain-like supercomputer switched on
hostage rescue, building intervention, desert The world’s largest supercomputer designed to
survival, medical aid and combat firing. work in the same way as the human brain has been
Ghost galaxy spotted next to Milky Way switched on for the first time. The ‘Spiking Neural
Scientists of the European Space Agency (ESA) has Network Architecture’ (SpiNNaker) machine is
discovered a massive and very faint galaxy near the capable of completing more than 200 million
©LegalEdge Tutorials Page 15 of 60
Replication or other unauthorized use of this material is prohibited by the copyright laws of India
actions per second. The SpiNNaker machine has Seth as the Madhya Pradesh High Court’s Chief
been designed and built in the University of Justice.
Manchester in the United Kingdom.
First ever plane with no moving parts launched 3. Justice AP Sahi- CJ of Patna HC
Scientists at the Massachusetts Institute of The President, under Article 217 (1) of the
Technology (MIT) have built and flown the first- Constitution of India, has appointed Justice
ever silent aeroplane with no moving propellers Amreshwar Pratap Sahi, former Judge of the
or jet turbines. The aeroplane is powered by an Allahabad High Court, to be the Chief Justice of
ionic wind technology. Ionic wind technology uses the Patna High Court.
a powerful electric field to generate charged ions,
which are then expelled from the back of the 4. Justice Sanjay Karol- CJ of Tripura HC
aircraft for generating thrust. Within the powers conferred under Article 217 (1)
Four new horned frogs discovered of the Indian Constitution, the President has
Four new species of horned frogs have been appointed Justice Sanjay Karol as the Chief Justice
discovered in Himalayan regions of Northeast of the Tripura High Court.
India, by a team of biologists from Delhi
University, University College Dublin and the 5. Justice AS Bopanna- CJ of Gauhati HC
National Museum (UK). The four new Indian The President, as per article 217 (1) of the Indian
species are Himalayan horned frog; the Garo Constitution, has appointed Justice Ajjikuttria
white-lipped horned frog; the Yellow spotted Somaiah Bopanna as the Chief Justice of the
white-lipped horned frog; and the Giant Gauhati High Court.
Himalayan horned frog.
China builds an ‘artificial sun’ 6. Justice DK Gupta- CJ of Calcutta HC
China has built an ‘artificial sun’ that reaches The President has appointed Justice Debasish Kar
temperature six times that of the core of the sun. Gupta as the Chief Justice of the Calcutta High
The reactor named ‘Experimental Advanced Court, as per Article 217 (1) of the Indian
Superconducting Tokamak’ (EAST) is designed to Constitution.
replicate the processes of the sun as part of a project
to turn hydrogen into cost-effective green energy. It 7. Justice VK Bist- CJ of Sikkim HC
involves fusion of hydrogen atoms together to form The President, Ram Nath Kovind, has appointed,
heavier elements, such as helium. Justice Vijai Kumar Bist as the Chief Justice of the
Definition of kilogram changed Sikkim High Court.
The world’s standard definition of the kilogram,
the ampere, the kelvin and the mole has been 8. Justice R Ranganathan– CJ of Uttarakhand HC
changed. The definition of the kilogram is based on Under Article 217 clause 1 of the Indian
the International Prototype of the Kilogram (IPK), Constitution, the President has appointed Justice
a cylinder of a platinum alloy stored at the Ramesh Ranganathan as the Chief Justice of the
International Bureau of Weights and Measures Uttarakhand High Court.
(BIPM) in France. The new definition of Kilogram
will be replaced by the Planck constant. 9. Justice Govind Mathur – Acting CJ of Allahabad
HC
The President has appointed Justice Govind
LEGAL AFFAIRS Mathur as the acting Chief Justice of the
Allahabad High Court after the retirement of
1. Justice NH Patil- CJ of Bombay HC Justice DB Bhosale.
Justice NH Patil is appointed as the Chief Justice
of Bombay High Court by the President of India, 10. Justice AS Dave- Acting CJ of Gujarat HC
under Article 217 (1) of the Indian Constitution. The President has appointed Justice AS Dave as
The Bombay High Court has strength of 94 Judges. the acting Chief Justice of the Gujarat High Court.
Also, there are two Chief Justices in the Bombay This appointment took place due to the transfer of
HC at present. Justice Akil Kureshi to the Bombay High Court.
2. Justice SK Seth- CJ of MP High Court
As per Article 217 (1) of the Indian Constitution, 11. 4 new Judges in the Supreme Court
the President has appointed Justice Sanjay Kumar As per Article 124 of the Indian Constitution,
President has appointed 4 new Judges in the

©LegalEdge Tutorials Page 16 of 60


Replication or other unauthorized use of this material is prohibited by the copyright laws of India
Supreme Court. They are- Justice Subhash Reddy,
Justice Hemant Gupta, Justice MK Rasikbhai and 17. Email address to report sexual harassment at
Justice Ajay Rastogi. workplace
The National Commission for the Women or the
12. AK Gupta- new Chairman of the Competition NCW has launched a separate email
Commission of India address,ncw.metoo@gmail.com which will
Former IAS officer, Ashok Kumar Gupta has been specifically deal with the issues and complains
appointed the new Chairman of the Competition related to the sexual harassment at the workplace.
Commission of India or the CCI. He will succeed The National Commission for Women Act, in the
Sudhir Mittal. He was appointed by the year 1992, created the NCW which is a statutory
Appointments Committee of the Cabinets or the body dealing with women’s rights and problems.
ACC which is chaired by the Prime Minister of
India. 18. General Public can visit the SC on Saturdays
The Chief Justice of India, Ranjan Gogoi, with the
13. 9th November- National Legal Services Day launching of the portal,
The National Legal Services Day is celebrated in http://guidedtour.sci.nic.in/redirect.drt
India on 9th November every year. It is to spread announced that the general public is now allowed
awareness among the public about the fair and to visit the Supreme Court on every Saturday from
just procedures of the legal services and provide 10 AM to 1 PM, by booking the tours on the above
free legal aid to the weaker strata of the society. mentioned portal.
The Supreme Court started it in 1995 to help the
poor and disabled. 19. Supreme Court held that long cohabitation leads
to presumption of marriage
14. Winter Session commences from 11th December The Supreme Court said that under Section 125 of
2018 CrPC, a man has to maintain his wife. This holds
The Parliament will begin its winter session from true for those couples also who cohabit for a long
11th December 2018 to and conclude it on 8th time without fulfilling the legal formalities of the
January 2019. Instead of being commenced in valid marriage.
November, it will start in December due to state
elections. 20. Meaza Ashenafi- First Female President of
Ethiopia’s Supreme Court
15. UP government renames 2 cities East African Women’s Rights activist, Meaza
The Uttar Pradesh government has approved the Ashenafi has been appointed the president of the
renaming of two cities, i.e. Allahabad as Prayagraj Supreme Court of Ethiopia. She is the first female
and Faizabad as Ayodhya. Now if this proposal of to become so. She is also the gender advisor at the
renaming is approved by the Union then the UN Economic Commission for Africa.
names will stand changed.
21. Pakistan Supreme Court held that Media Trials
16. Ordinance to amend the Companies Act, 2013 is cannot be allowed
promulgated The Pak Supreme Court held that media trials
As per Article 123 of the Constitution of India, the cannot be allowed in Pakistan. It also issued some
President has given his assent to the promulgation directions to the PEMRA or Pakistan Electronic
of the ordinance which aims to amend the Media Regulatory Authority to properly regulate and
Companies Act of 2013. The main objective of this supervise media trials in sub-judicial issues.
amendment is to encourage ease of doing business
in India and betterment of the compliance levels.

©LegalEdge Tutorials Page 17 of 60


Replication or other unauthorized use of this material is prohibited by the copyright laws of India
LE Explains

Disaster Managenment Act and Institutes in India

INTRODUCTION:- constituted by the State Authority, which is responsible


A disaster management cell under the aegis of Ministry to coordinate and ensure compliance of the guidelines.
of Agriculture was established in furtherance of the UN
General Assembly’s declaration of the decade of 1990s as  DISTRICT DISASTER MANAGEMENT
'International Decade for Natural Disaster Reduction'. AUTHORITY (DDMA):-
With an aim to review the existing arrangement for State Government via a notification in the state budget is
preparedness and mitigation of natural disasters a High responsible for the establishment of DDMA. The collector
Powered Committee (HPC) was constituted in the year or district magistrate or Deputy Commissioner is
1999. The Ministry of Home Affairs was conferred with appointed as the chairman.
the control of the disaster management division of
Ministry of Agriculture in the year 2002. In furtherance of establishing an effective and able regime
Until 2005 India did not have a disaster management law. to manage disasters the following institutions have been
It was in 2003 that Gujarat government enacted Gujarat established.
Act for disaster management. As per the
recommendations made by the HPC in its report 1. National Disaster Response Force (NDRF)
submitted on October 2001, the Government enacted the National Disaster Response Force is constituted with an
Disaster Management Act on 23rd December, 2005. The aim to provide specialist response in situations of
disaster Management Act implemented in the year 2005 disasters. NDMA is conferred with the power to exercise
aims to ensure effective disaster management. The act control over NDRF.
introduces a three tier institutional structure for better
2. National Disaster Response Fund
implementation of the act. The three tier structure that
As per the provisions of the Disaster Management Act
decentralizes power at National, state and District level
2005 the central government is required to constitute a
has been formulated. As per the provisions of the act
National Disaster Response Fund. It is pertinent to note
National Disaster Management Authority, State Disaster
that as per the recommendations of the 13th Finance
Management Authority, District Disaster Management
Commission the erstwhile National Calamity
Authority and disaster related funds have been
Contingency Fund (NCCF) which was instituted as per
established.
the recommendations of the 11th finance commission has
NATIONAL DISASTER MANAGEMENT been merged with the National Disaster Response Fund.
AUTHORITY (NDMA):-
The central government as per the provisions of the 3. National Disaster Mitigation Fund:-
Disaster Management Act is mandated to establish a As per the provisions of the Disaster Management Act
National Disaster Management Authority. It is 2005 the central government is required to institute a
established to deal with all types of disasters and has the National Disaster Mitigation Fund. Money in the
responsibility to lay down guidelines and policies on following fund is exclusively used for the purpose of
disaster management. Prime Minister is the ex-officio mitigation.
chairperson of the NDMA. He has the prerogative to call IMPORTANT POINTS:-
for meetings as and when he thinks fit. The NDMA can 1. As per section 75 of the Disaster Management Act the
have maximum of 9 members, nominated by the central government has the power to make rules for the
chairperson. implementation of the provision laid down in the Act.
National Executive committee is constituted to assist
NDMA and that the committee is responsible for 2. To promote a culture of risk awareness International
formulating the Nation Plan (Disaster Management Plan) Day for Disaster Reduction is observed on 13th of October.
for the entire country. Theme for 2018 is ‘Reducing Disaster Economic Losses’

STATE DISASTER MANAGEMENT AUTHORITY 3. National Awards in the name of Netaji Subhash
Chandra Bose to honor people involved in disaster
(SDMA):-
response operations have been announced. Netaji
Every state government establishes a State Disaster
Subhash Chnadra Bose declared the formation of Azad
Management Authority with Chief Minister as the
Hind Fauz on 21st October 1943; the awards have been
chairperson of SDMA. A State Executive Committee is
instituted to celebrate the 75th anniversary of the same.

©LegalEdge Tutorials Page 18 of 60


Replication or other unauthorized use of this material is prohibited by the copyright laws of India
General Elections in Indian Constitution
As per the provisions laid down in the Indian After the constitutional amendment in the year 1993
Constitution General Elections are held every five years election commission permanently became a multi
in India. General elections are held for Lok Sabha and member body.
various State Legislative Assemblies. After having
received Independence in the year 1947, the first General Note:-In furtherance of the directive issued by the
Elections in India were held in the year 1951-52. Elections Supreme Court Election Commission of India has
in the year 1951-52 were conducted for 489 and removed NOTA (None of the above) option from the
represented 26 states. ballot papers of Rajya Sabha and Legislative council
The first General Elections were won by the Indian elections. However the option will still be available in
National Congress and Jawahar Lal Nehru was elected as Lokm Sabha and state assembly elections.
the first Prime Minister of India. Shri G.V. Mavalankar The option was introduced in the year 2013 in 4 state
presided as the speaker of the first Lok Sabha. assembly elections namely Chhattisgarh, Mizoram,
Lok Sabha consists of representatives who are elected by Rajasthan and Madhya Pradesh. The first general
the people through direct election on the basis of the adult elections to see nation wise use of NOTA was the 2014
suffrage. As per the provision enshrined in the Lok Sabha elections.
constitution the maximum strength of the House can be
552 members. The composition enshrined is bifurcated as EVM (ELECTRONIC VOTING MACHINE):- With the
530 seats designated for the representation of the States, advancement in technology Electronic voting machines
up to 20 seats for the representation of the Union were introduced to improve election procedures. EVM’s
Territories and not more than two members for the were first used in the year 1982 in Kerala for legislative
representation of the Anglo-Indian Community which assembly elections.
are to be nominated by the President, if in the opinion of
the president the community does not have adequate PARTIES IN THE ELECTION:- It is necessary for all
representation in the house. political parties which wish to contest elections to register
Currently 16th Lok Sabha is in session and the next themselves with the Election Commission of India.
General Elections will be held in the year 2019 to elect the Political Parties in India can be divided into three broad
17th Lok Sabha. categories namely National parties, State recognized
Party and registered unrecognized parties. Election
ELECTION COMMISSION:- commission of India after having followed the eligibility
Elections are conducted as per constitutional provisions criteria is responsible for according different status to
and legislative enactments. Part XV of the Indian political parties.
constitution lays down provisions pertaining to elections At present 7 parties have been accorded the status of
in India. Article 324(1) talk about the establishment of a National Parties namely:-
commission responsible for the superintendence, Bharatiya Janata Party (BJP), Congress, Bahujan Samaj
direction and control of the entire election process. Party (BSP), Nationalist Congress Party (NCP),
Election commission is headed by the chief election Communist Party of India (CPI) Communist Party of
commissioner who is appointed by the President for a India-Marxist (CPM) and All India Trinamool Congress.
term of six years or till he attains the age of 65, whichever
is earlier. The Chief Election Commissioner is appointed 2014 GENERAL ELECTIONS:- 2014 Lok Sabha
in accordance with Article 324(2) of the Indian elections were conducted to constitute 16th Lok Sabha and
constitution. Sukumar Sen was the first chief election members to 543 parliamentary constituencies were
commissioner. Currently the post is held by H.S. Brahma. elected. The election was held in 9 phases and thus made
He is succeeded Nasim Zaidi. it the longest election in the country’s history. The results
There was only one Chief Election Commissioner from were declared on 16th May 2014 and National Democratic
1947 to 16th October 1989. It was in the year 1990 that two Alliance was voted to power.
election commissioners, with aim to reduce the work load
of chief election commissioner were appointed.

©LegalEdge Tutorials Page 19 of 60


Replication or other unauthorized use of this material is prohibited by the copyright laws of India
Potpourri

20 LEGAL ONE LINERS


1. Criminal procedure is a subject of Concurrent list. 13. Child Welfare Committee is constituted under
2. Arrest means total restraint and complete section 27 of the Juvenile Justice (Care and
deprivation of liberty. Protection of Children) Act, 2015.
3. General exceptions are contained in Chapter IV of 14. Juvenile Justice Fund is to be created by the State
IPC. Government under section 105 of the Juvenile
4. Minimum persons required for committing the Justice (Care and Protection of Children) Act, 2015.
offence of criminal conspiracy are two. 15. The appropriate writ issued by the Supreme Court
5. Consent of minor is wholly immaterial in case of to quash the appointment of a person from a
the offence of kidnapping. public office is Quo Warranto.
6. Indian Evidence Act, 1872 was drafted by Sir 16. The words “Secular” and “Socialist” were inserted
James F. Stephen. in the Preamble of the Constitution of India by the
7. Writ of Mandamus cannot be issued under Article 42nd Amendment.
32 where fundamental right is infringed by a 17. “What cannot be done directly, cannot be done
private body. indirectly”. This statement is related to Doctrine of
8. Fundamental Duties as provided in the Colourable Legislation.
Constitution have been added on the 18. The concept of Fundamental Duties is derived in
recommendation of the Santhanam Committee. Indian Constitution from the Constitution of
9. An agreement in restraint of trade is void. U.S.S.R.
10. Pigeon hole theory was propounded by Salmond. 19. Whether a Bill is Money Bill or not, is to be
11. Remedy of Law of Torts is Unliquidated Damages. decided by the Speaker of the Lok Sabha in case of
12. If a person is held liable for the act/omission the Union Legislature.
/torts of any other person, it is known as vicarious 20. Under a contract between the parties, plea of
liability. undue influence may be raised only by the party
whose consent was so caused.

Maxims and Foreign Phrases


1. Turpis Causa- immoral cause violent crowd following the orders of his superior,
#It is used to refer immoral, illegal or baseless his actions will be ex cathedra.
considerations underlying a contract which will 4. In situ: In its place.
have the effect of rendering the contract void (not # It is a Latin phrase that translates literally to "on
binding between the parties). The contract was for site “or "in position". It means "locally", "on site",
the purpose of paying bribe to a public servant "on the premises" or "in place" to describe an event
and thus is turpis causa. where it takes place, and is used in many different
2. Lex Fori- law of the forum contexts.
# It is used to indicate that the law applicable 5. Consensus ad idem- agreement as to the same
must be that of the country (or state or thing in the same manner.
jurisdiction) where the suit is bought and not that # Monica has a cat named Rosita and she has also
of country where the act was committed. The term named her BWM car as Rosita. Monica agrees to
is relevant in international law matters where act sell her Rosita to Lisa, believing that she is selling
was committed in one country but case is being her cat. On the other hand Lisa is in a belief that
bought in another country. Lex fori will govern the she is buying Monica’s BMW car. Here both the
procedural aspects of how the court operates parties are not agreeing to the same thing and they
while the substantial rights of the parties will be have no consensus ad idem. It is used in contract
determined as per the law of the country where law. Contracts without consensus ad idem are
the act was committed. void in nature.
3. Ex cathedra: With official authority. 6. Bona Vacantia- Latin Phrase meaning ownerless
# With the full authority of office (especially that goods.
of the Pope, implying infallibility as defined in # It is a common law practice brought in India by
Roman Catholic doctrine). In present context any Britishers. Under Indian law if a person dies
police officer using lathi charge to disperse the without any heirs, his property becomes the
property of state.

©LegalEdge Tutorials Page 20 of 60


Replication or other unauthorized use of this material is prohibited by the copyright laws of India
7. Eminent Domain- Right of the government to with the terms and conditions of the treaty. One
expropriate private property for public use. cannot just create a contract with no intention of
# Government proposes the construction of complying to the clauses.
railway tracks in the area. Mr. X’s farm is in 9. ad quod damnum: According to the harm.
between the proposed track. In such case # Used in tort law. Implies that the reward or
government can ask Mr. X to relocate (by paying penalty ought to correspond to the damage
just and fair compensation) and would take his suffered or inflicted.
property for constructing the railway track. 10. animus nocendi: intention to harm
8. Pacta sunt servanda- Latin phrase meaning #The subjective state of mind of the author of a
agreements must be kept. crime, with reference to the exact knowledge of
# Mostly used in respect of international law. Eg. illegal content of his behaviour, and of its possible
If a group of countries sign up a treaty or an consequences. (mens reus)
agreement then each signatory should comply

KEY DIFFERENCES BETWEEN IMPORTANT CONCEPTS


1. Coercion and Undue Influence this disciple for the agreement to transfer
(a) In coercion the consent to an agreement is property.
obtained by committing or threatening to
commit an offense forbidden by the Indian 2. Quasi Contracts and Contracts
Penal Code. In undue influence the consent is (a) A Contract is based on the mutual will and
obtained by dominating the will of the other consent of the parties to a contract whereas a
person. quasi contract is one which resembles a
(b) In coercion, there is a threat or actual use of contractual relationship and arises in
violence for obtaining the consent for the situations where one party gets unjust
agreement while in undue influence passive enrichment.
force by way of mental or moral influence is (b) In a contract the obligations of the parties are
used to obtain consent of the party to the as set out in the contract whereas in cases of
agreement. quasi contract this basis of the obligation is
(c) In coercion, the party threatening the other that no one should have unjust benefit at the
party may be a stranger but in undue cost of the other. A person who is enriched by
influence, there exists a relationship between a benefit at the expenses of another is bound
the parties where one party is in a dominating to compensate the other party for the benefit
position. received.
(d) In cases of coercion, there is a presumption (c) A contract is an agreement while a quasi
that there is no coercion and the burden of contract is not.
proof is on the party alleging coercion but on (d) For a contract to be valid the essential
the other hand in cases of undue influence the elements such as free consent, lawful object
burden of proof is on the person in the and consideration must be fulfilled while in
dominating position that the consent of the quasi contracts, the essential elements for a
other party was taken freely without any contract need not be fulfilled for a quasi
undue influence. contract to be enforceable.
(e) Coercion also attracts criminal liability and is (e) For e.g. if A supplies food to B, a lunatic, A is
punishable under the provisions of the Indian entitled to be reimbursed from B’s property
Penal Code while undue influence is not even in the absence of a contract since B has
considered an offence under the Indian Penal received a benefit and hence B is under a
Code. quasi-contractual obligation to compensate A
(f) For e.g. when a person puts a gun to the head for the benefit received. On the other hand, if
of another and asks him to sign a contract, it is A and B have a contract where A will supply
an example of coercion. But if a spiritual guru B with food, the obligation of B to compensate
knowing his influence over his disciple makes A for the food arises from the contract
the disciple sign an agreement transferring a between the parties.
property to him, the spiritual guru has
exploited his dominant position and has used 3. Joint and Several Tortfeasors
undue influence in obtaining the consent of

©LegalEdge Tutorials Page 21 of 60


Replication or other unauthorized use of this material is prohibited by the copyright laws of India
(a) When two or more persons act together to accordance with the proportion of their
commit a tort against a person causing a single contribution to the commission of the tort.
injury they are said to be joint tortfeasors. On However in cases of several tortfeasors, each
the other hand, when two or more persons act tortfeasor will be independently responsible
independently and different damages are for the tort committed by him and not for the
caused to a person, they are said to be several torts committed by the other tortfeasors.
tortfeasors. (e) In cases of joint tortfeasors, each joint
(b) In joint tortfeasors, all parties must actively tortfeasor is jointly and severally liable for the
participate together and there must be a damages and therefore the victim can claim
common goal. On the other hand in several the full damages from any one of the joint
tortfeasors the parties act independently and tortfeasors however, where one tortfeasor
there is no common goal or design. pays more than his share of the damages, he is
(c) The cause of action for joint tortfeasors is the entitled to recover the amount from the other
same for each of the joint tortfeasors while the tortfeasors on the principle of contribution.
cause of action for several tortfeasors is This is not possible in cases of several
different. tortfeasors.
(d) In cases of joint tortfeasors, the damages must
be borne by each of the tortfeasors in

©LegalEdge Tutorials Page 22 of 60


Replication or other unauthorized use of this material is prohibited by the copyright laws of India
LE Prep Assist
Mentor View- General Knowledge
CLAT esa lkekU; Kku oxZ dh rS;kjh dSls djsa\
gj rS;kjh ds dqN i{k gksrs gS] ftuds ckjs esa tkuuk lcls t:jh gS A igyk i{k bl izfdz;k es gS]+ Þle;ß vkSj nwljk i{k gS fo’k; (subject
content) A
fo’k;oLrq %& lkekU; Kku esa izeq[kr% nks vyx&vyx i{kks ij /;ku nsuk gS A
1 Lkkef;dh (Current Affairs) :- lkef;dh es izeq[kr%
 jk’Vªh; lekpkj (National Affairs)
 varjk’Vªh; lekpkj (International Affairs)
 f}i{kh; ekeys (Bilateral Affairs)
 [ksy (Sports)
 pfpZr txgsa (Place in News)
 pfpZr O;fDr (Person in News)
 gkfy;k ?kVuk,~W (Recent Happenings)
 IkqjLdkj vkSj vU; ?kVukdze (Awards and Other Event)
 varfj{k vkSj j{kk (Space & Defense)
Current Affairs dh RkS;kjh esa lcls igys ;g tkuuk t:jh gS fd ÞD;k djuk gS\ß
Online Pollution ls nwj jgs%&¼vki 5&6 pqfuUnk Websites dh txg flQZ ,d fo”oluh; (Authentic) Website ls fy[ks vkSj mlh ls
i<sA
fy[ks gq, Current Affairs dks jkstkuk i<s vkSj g¶rs esa mls fQj ls nksgjk,A
feys gq, Test Papers dks utjvankt u djsa- vkSj MonthLE dks Hkh nksgjk,~WA
2 LkkekU;Kku (Static G.k):- bl oxZ esa gesa dqN fo”ks’k fo’k;ksa is /;ku nsuk gS
 bfrgkl (History)
 Hkwxksy (Geography)
 lkekU; foKku (Economics)
 fofo/k (Miscellaneous)
fo’k;&oLrq (Subject Content) dh rS;kjh ds ;g tkuuk t:jh gS fd bfrgkl] Hkwxksy] LkkekU; foKku vkSj vFkZ”kkL=] d{kk&10oha rd dh
NCERT ds Lrj rd vkrk gSA
blfy, “RSM” ,d egRoiq.kZ स्रोत gS vkidh Static G.k. dh rS;kjh dk A mlds vykok vki Theory i<us ds ckn mlds iz”uks dks gy djus esa
?;ku yxkb, vkSj mlds fy, Lucent (objective) vkSj Topic test tks LE ds iksVZy esa gS] mlds iz”uks dk vH;kl djsaA ckr vxj fofo/k
(Miscellaneous) dh djsa rks iqLrds ys[kd egRoiw.kZ iqjLdkj, j{kk, fQYes,a [ksydwn varfj{k foKku vkSj rduhdh vkSj vU; lkekU; tkx:drk ds
fo’k; blds vUrxZr vkrs gS] ftudh rS;kjh Hkh RSM Lucent (objective) vkSj topic test vkSj Question sheet ls gksuh pkfg, A
vkSj vc ckr le; dh rks igys gh ckr gks pqdh gS fd lkef;dh (Current Affair’s) esa fnu Hkj dh xfrfof/k D;k gksuk pkfg,A jgh ckr
General Knowledge dh rkss mls gj nwljs & rhljs fnu i<rs jguk] iz”u&i= djuk vkSj nksgjkuk lcls vPNh j.kuhfr gksxhA
RkS;kjh ds dqN egRoiq.kZ fcanq %&
 lkekU; Kku dks fu;fer i<uk] mlds iz”uks dks gy djuk vkSj nksgjkrs jguk t:jh gS A
 fdlh Hkh ,d oxZ pkgs og lkef;dh (Current Affair’s) ;k Static gks T;knk le; ;k esgur djuk xyr gS A CLAT dh ijh{kk
vizR;kf”kr:Ik ls bues cnyko djrh gS A
 de fdrkcksa ls T;knk ckj i<uk lcls t:jh gS A
Shubham Dwivedi
SME (General Knowledge)
LegalEdge, Bhopal

©LegalEdge Tutorials Page 23 of 60


Replication or other unauthorized use of this material is prohibited by the copyright laws of India
Mentor View- Mathematics
Dear CLAT Aspirants,
You are now well past the midway of your CLAT prep journey. I hope you all are doing well but in case you have
lost the plot, especially in math (quiet obviously), then its high time that you buckle up because Math, though
worth only 20 marks on paper, can be a game-changer. Its lesser proportion in the overall marks distribution won’t
take away anything from the fact that it still requires the same or sometimes even more time to comprehend than
the other topics might take.
Few questions are very frequently asked to me and what follows is my take on them I hope you will find it useful.

My basics are weak! – This shouldn’t be an excuse for it to remain so anymore. Some might feel the need to study
NCERT books again but that’s impractical knowing the time it would take. Just mark the steps where you missed
the plot of the answer in your class notes or the book that you are referring to, and understand it later with the help
of the subject expert. Keep those concepts written in the book so that it strikes you every-time you revise.
This answer has more to do with your psychology than anything else. If you feel ‘Math isn’t my cup of tea’ and
searching for a crash method that would just somehow fetch you 5-10 marks thinking that would be enough than
believe that you deserve more. Scoring better won’t become a physical reality unless you feel and act that way.

How much is enough? – A Book? 100 Questions per topic? To be honest, there is no fixed number as who knows
whether the 100 questions that you solved actually covered the whole difficulty spectrum of the topic or not. Those
who are good at Math sometimes fall prey to the mentality of cherry picking the most difficult questions. Just as a
batsman takes a few balls to first settle down and feel comfortable at the pitch (even if it’s a T-20) and makes it large
very artistically playing every ball on its merit similarly go for all the questions of a recommended book and
categorize them as LOD (Level of Difficulty) 1,2 & 3. It’s only after you have done all the questions once that you
can have the luxury of solving only a few LOD 3 questions just before the exam when you won’t be left with much
time.

Not scoring as much as I study! - “Study More” would be an obvious answer but there’s more to it. It’s a vague
question and no one actually has the answer to it except you and if even you don’t have a clue then go for your
Mock Analysis. Even when CLAT has gone offline, we have still kept a good number of online tests because nothing
can beat the analysis that they provide you. When you do that analysis you might understand how doing lesser
questions with more accuracy is better than doing more questions but with lesser accuracy. The following table
would illustrate it better:
Attempts Correct Wrong Accuracy Final Marks
18 10 8 56% 8
13 9 4 70% 8
11 10 1 91% 9.75
Increasing study hours is always good but those must be utilized properly. In Math you might be wasting a lot of
time in calculations. Stick a few sheets on your room walls that has tables of ‘squares and cubes of first 30 numbers’,
‘important percentages’, ‘Algebra formulae’ etc that you might be wasting time at doing every time that you
encounter then in your mocks. Also, streamline your preparation in accordance with the questions asked in
previous year papers.

I keep forgetting previously studied topics – Isn’t that how it should be? What did you expect? Studying a topic is
not a one-time investment. Go old school. Keep all important theorems about a topic written in a nutshell
somewhere and keep revising it with LOD 3 questions marked in the book. How much that revision content should
be? As much that you can revise within 15 minutes so that revising it does not become a burden for you.
Abhishek Bagga
SME (MAthematics)
LegalEdge, Bhopal

©LegalEdge Tutorials Page 24 of 60


Replication or other unauthorized use of this material is prohibited by the copyright laws of India
Prep Test– Logical Reasoning
1. Read the information given below and answer the question that follows.
Six students A, B, C, D, E and F are sitting in the field. A and B are from Nehru House while the rest belong to
Gandhi House. D and F are tall while the others are short. A, C and D are wearing glasses while the others are not.

Which two students, who are not wearing glasses, are short?
(a) A and F (b) C and E (c) B and E (d) E and F

2. Read the information given below and answer the question that follows.

Six students A, B, C, D, E and F are sitting in the field. A and B are from Delhi while the rest are from Bangalore. D
and F are tall while others are short. A, C and D are girls while others are boys. Which is the tall girl from
Bangalore?
(a) C (b) D (c) E (d) F

3. Read the information given below and answer the question that follows.

On a shelf are placed six volumes side-by-side labelled A, B, C, D, E and F. B, C, E, and F have green covers while
others have yellow covers. A, D and B are new volumes while the rest are old volumes. A, C, B are law reports
while the rest are medical extracts. Which two volumes are old medical extracts and have green covers?
(a) B, C (b) C, D (c) C, E (d) E, F

4. Read the information given below and answer the question that follows.

In a group of six women, there are four dancers, four vocal musicians, one actress and three violinists. Girija and
Vanaja are among the violinists while Jalaja and Shailja do not know how to play on the violin. Shailja and Tanuja
are among the dancers. Jalaja, Vanaja Shailja and Tanuja are all vocal musicians and two of them are also violinists.
If Pooja is an actress, who among the following is a dancer, vocal musician and a violinist?
(a) Jalaja (b) Shailja (c) Tanuja (d) Pooja

5. Answer the question based on the information given in the passage.

The city administration has decided to impose an entry tax on cars that enter the city during business hours, that is
9 am to 6 pm. The tax is greater than the cost of round-trip bus or monorail fare from nearby points, and the
administration believes that such a move would push people to take up the public modes of transportation and this
will ultimately reduce the congestion on the roads in the new 6 months.

From the following statements, identify the ones that weaken the administration’s claim:

I. the price of petrol and diesel is expected to fall substantially in the coming months and reduce costs of driving
your own vehicle.
II. the existing fleet of buses and monorail already operate near full capacity and can hardly accommodate new
passengers.
III. 10% of the traffic in the city is made up of cars that enter the city everyday and on average, about 2% of the cars
running in the city are added to the city traffic by virtue of new purchases by city residents every 6 months.

(a) I & II (b) II & III (c) I & III (d) All of the above

6. In the question a statement is given followed by two arguments numbered I and II. Mark the answer as:
Statement: All living below the poverty line should be given jobs.
Arguement I. Yes, else they will die of malnutrition.
Arguement II. No. They might become lazy.

(a) If only argument A is strong (b) If only argument B is strong


(c) If both arguments are strong (d) If neither of them are strong

©LegalEdge Tutorials Page 25 of 60


Replication or other unauthorized use of this material is prohibited by the copyright laws of India
7. In the question a statement is given followed by two arguments numbered I and II. Mark the answer as:
Statement: Dowry should be abolished

Arguement I. Yes. It is becoming unfashionable.


Arguement II. No. It gives a boost to the economy
(a) If only argument A is strong (b) If only argument B is strong
(c) If both arguments are strong (d) If neither of them are strong

8. Solve the following question and mark the most appropriate option.
In the following series of numbers, find out how many times 1, 3 and 7 have appeared together, 7 being in the
middle and 1 and 3 on either side of 7?

2931737771331738571377173906
(a) 3 (b) 4 (c) 5 (d) None of these

9. Solve the following question and mark the most appropriate option.

How many even numbers are there in the following sequence of numbers each of which is immediately followed by
an odd number as well as immediately preceded by an even number?

86788932753422355228119
(a) One (b) Three (c) Five (d) None of these

10. Solve the following question and mark the most appropriate option.
How many such 6's are there in the following number series, each of which is immediately preceded by 1 or 5 and
immediately following by 3 or 9?

263756429613416391569231654321967163
(a) Nine (b) One (c) Two (d) Three

11. Solve the following question and mark the most appropriate option.
In the following series, how many such odd numbers are there which are divisible by 3 or 5, then followed by odd
numbers and then also followed by even numbers?

12, 19, 21, 3, 25, 18, 35, 20, 22, 21, 45, 46, 47, 48, 9, 50, 52, 54, 55, 56

(a) Three (b) One (c) Two (d) None of these

12. Solve the following question and mark the most appropriate option.

If all the alphabets are dropped from the above arrangement, which of the following will be sixth to the left of 12th
from the left end?
(a) © (b) (c) # (d) None of these

13. Solve the following question and mark the most appropriate option.

Four of the following five are alike in a certain way based on their positions in the above arrangement and so form a
group. Which is the one that does not belong to that group?
(a) ∇%I (b) QD3 (c) FQ∎ (d) P6M

14. Solve the following question and mark the most appropriate option.

Which of the following is eighth to the left of fifteenth element from the right end of the above arrangement?
(a) * (b) ∇ (c) C (d) None of these

©LegalEdge Tutorials Page 26 of 60


Replication or other unauthorized use of this material is prohibited by the copyright laws of India
15. Solve the following question and mark the most appropriate option.

How many such consonants are there in the above arrangement each of which is immediately preceded by a
symbol?
(a) One (b) Two (c) Three (d) None of these

16. Solve the following question and mark the best possible option.
If 'INTELLIGENT' is called for a secret message to be teleprinted as 'FKQBIIFDBKQ', how is the reply
'ENTERTAINMENT' to be relayed?
(a) BKQBQOFKXJBKQ (b) BQJXOKQOFJBKQ
(c) BKQBOQXFKJBKQ (d) None of these

17. Solve the following question and mark the best possible option.
In a certain code 'BREAKTHROUGH' is written as 'EAOUHRBRGHKT'. How is 'DISTRIBUTION' written in that
code?
(a) TISTBUONDRII (b) STTIBIJONRIDI (c) STTIBUDIONRI (d) None of these

18. Solve the following question and mark the best possible option.
Z is maternal uncle of Y, X is maternal grandfather of Z. S is grandson of X. How is S related to Y?
(a) Maternal grandfather (b) Maternal uncle
(b) Cousin (d) None of the above

19. Solve the following question and mark the best possible option.
P's father is Q's son. M is the paternal uncle of P and N is the brother of O. How is M related to N?
(a) Nephew (b) Cousin (c) Data inadequate (d) None of these

20. Solve the following question and mark the best possible option.
D is the son-in-law of B, who is the brother-in-law of A, who is the brother of C. How is A related to B?
(a) Father (b) Son (c) Data inadequate (d) None of these

21. Solve the following question and mark the best possible option.
P is the only son of O who is the only daughter of S. D is the mother of B and daughter-in-law of O. How is B
related to O?
(a) Grandson (b) Grand-daughter (c) Son (d) Cannot be determined

22. Solve the following question and mark the best possible option.
S's father is Q's son. R is the uncle of S and P is the brother of Q. How is P related to R?
(a) Uncle (b) Son (c) Brother (d) Cannot be determined

23. In the question, some symbols are used which define the relationship between two categories. Using those, solve the
question and mark

(a) Z – Y = Z (b) X * Y – Z (c) X ∆ Y Φ Z (d) None of these

©LegalEdge Tutorials Page 27 of 60


Replication or other unauthorized use of this material is prohibited by the copyright laws of India
24. In the question, some symbols are used which define the relationship between two categories. Using those, solve the
question and mark

(a) X × Y – Z (b) X - Y × Z (c) X + Y × Z (d) X × Y = Z

25. In the question, some symbols are used which define the relationship between two categories. Using those, solve the
question and mark

X - Y + Z implies:
(a) X + Y + Z (b) X Φ Y – Z (c) X - Y Φ Z (d) None of these

26. Solve the following question and mark the best possible option.

X - Y = Z implies
(a) X + Y - Z (b) X Δ Y × Z (c) X + Y Φ Z (d) X + Y × Z

27. In the question, some symbols are used which define the relationship between two categories. Using those, solve the
question and mark

X = Y - Z does not implies:


(a) Y Φ X = Z (b) X Φ Y Φ Z (c) Z Φ X = Y (d) None of these

28. Read the following statement considering them to be true and therefore judge their conclusions numbered I, II and
III and given your answer accordingly.
Statements: All bananas are apples. Some apples are oranges. All oranges are grapes.
Conclusions: I. Some grapes are bananas.
II. Some grapes are apples.
III. Some oranges are bananas.
(a) one follows (b) Only I follow (c) Only II follows (d) I and II follow

©LegalEdge Tutorials Page 28 of 60


Replication or other unauthorized use of this material is prohibited by the copyright laws of India
29. Read the following statement considering them to be true and therefore judge their conclusions numbered I, II and
III and given your answer accordingly.
Statements: Some tables are chairs. All chairs are benches. All benches are desks.
Conclusions: I. Some desks are table
II. Some benches are tables.
III. Some desks are chairs.
(a) Only I follow (b) Only II follows (c) Only III follows (d) All follow

30. Read the following statement considering them to be true and therefore judge their conclusions numbered I, II and
III and given your answer accordingly.
Statements: Some towns are villages. Some villages are lanes. Some lanes are hamlets.
Conclusions: I. Some hamlets are villages. II. Some lanes are towns. III. Some hamlets are towns.
(a) Only III follows (b) Only I follow (c) Only II follows (d) None follows

31. Read the following statement considering them to be true and therefore judge their conclusions numbered I, II and
III and given your answer accordingly.
Statements: All buildings are mirrors. Some mirrors are pens. No pen is paper.
Conclusions: I. Some papers are buildings. II. Some pens are buildings. III. Some papers are mirrors.
(a) None follows (b) Only I follow (c) Only II follows (d) Only III follows

32. Read the information given below and answer the question that follows.
Mohsin is carrying 8 fruits — orange, mango, pineapple strawberry, guava, blueberry, apples and watermelon to
the farmhouse. However, he has only two boxes of equal weight and each will carry same number of fruits.
Therefore, he has to ensure:-
A. orange and pineapple will be together, but not with watermelon.
B. Mango and strawberry cannot be with pineapple.
C. Blueberry will be in the same box with orange and guava but not with the apple. One more fruit will be with
blueberry.

If apple and watermelon are in the same box which are the other two fruits in the same box?
(a) Strawberry and pineapple (b) Mango and guava
(c) Mango and Strawberry (d) Cannot be determined

33. Read the information given below and answer the question that follows.
Seven army recruits Laxman, Mahender, Narender, Omprakash, Preetam, Qutab and Raman are standing in a
column for physical inspection in order of height, shorter first. Omprakash is standing between Laxman and Qutab.
Narendar is taller than Laxman. Raman is shorter than Narendar. Mahendar and Preetam are not the two tallest.
Preetam is two places ahead of Qutab. Which of the following is true?
(a) Narendar is the tallest (b) Qutab is taller than Laxman
(c) Raman is ahead of Omprakash (d) Mahendar is the shortest

34. Read the information given below and answer the question that follows.
In a small organization Saurav and Kartik plays tennis and basketball very well. Manpreet and Lokesh are very
good in athletics, tennis and swimming. Kartik and Nikhil are good in athletics and badminton. Ganesh and Lokesh
are good in badminton and hockey. Prakash and Rohit are good in Hockey and basketball Ganesh, Prakash, and
Saurav are good in swimming. Manpreet and Nikhil also play hockey well. If an inter-organizational event is
arranged and only one person can be selected from one organization, who will be selected from this organization if
tennis, Swimming and badminton has to be played in the competition?
(a) Saurav (b) Kartik (c) Manpreet (d) Lokesh

©LegalEdge Tutorials Page 29 of 60


Replication or other unauthorized use of this material is prohibited by the copyright laws of India
35. Read the information given below and answer the question that follows.
One day five friends Amar, Bitu, Chetan , Darshan and Ehsaan having bikes Pulsar, Splendor, Apache, Unicorn and
Hayabusa but not in the same order decided to race. Chetan came second last and was not driving Pulsar or
Hayabusa. Ehsaan came just after Amar and was driving Unicorn. The person who came last was driving Apache.
Darshan did not come last and was driving Hayabusa. Amar had neither came first nor last and was not driving
Apache or Splendor. Who was driving Splendor?
(a) Darshan (b) Amar (c) Chetan (d) Bitu

36. Read the information given below and answer the question that follows.
Six friends – A, B, C, D, E and F – are sitting in a row such that all of them are facing in the same direction. A and F
are friends and hence, are sitting together. D is sitting at one of the extreme ends. C is sitting to the immediate right
of B. E is sitting next to A and D. Who among the following is sitting at the other extreme end?
(a) B (b) C (c) A (d) Cannot be determined
37. Read the information given below and answer the question that follows.
A team of six members is selected from a bunch of boys B1, B2, B3, B4, B5 and B5 and four girls G1, G2, G3 and G4.

Some of the criteria are as follows


B2 and B4 have to go together
B3 cannot go with G2
B6 cannot go with the B1 and B4
G1 and G3 have to be together
G1 cannot go with G4
B1 and B5 have to be together

If four of the members including B5 have to be boys and one of the girls has to be G3 then the team members are:-
(a) B1, B4, B5, B6, G1, G3 (b) B2, B4, B5, B6, G1, G3
(c) B1, B2, B4, B5, G3, G4 (d) B1, B2, B4, B5, G1, G3

38. Solve the following question and mark the best possible option.
In a certain code 'nee tim see' means 'who is it', 'ble nee see' means 'where is it', 'ble see tee' means 'where is what'.
What is the code for 'who is where'?
(a) tim ble see (b) tim nee see (c) tim ble nee (d) nee ble see
39. Read the information given below and answer the question that follows.
Its dinner time and snow white is serving soup to the seven dwarfs namely grumpy, happy, bashful, dopey, sneezy,
sleepy and dreamy. The dwarfs are standing in a line where sneezy is between bashful and dopey. Grumpy is just
before happy and dreamy is just before sleepy. Bashful is not the last and he is behind sleepy. Dopey is also not last.
Who is standing just before grumpy?
(a) sleepy (b) dreamy (c) happy (d) dopey

40. Solve the following question and mark the best possible option.
Mac is brother of Tina and brother-in-law of Tim. Sheron has two children and her son-in-law is Tim, Sheeba's
grandmother is Sharon. Teena is Sheeba's aunt. Which of the following statements is true?

(a) Sheeba is daughter of Tim (b) Teena is Tim's sister-in-law


(c) Sheeba is niece of Tim (d) Sheeba is niece of Mac

©LegalEdge Tutorials Page 30 of 60


Replication or other unauthorized use of this material is prohibited by the copyright laws of India
Prep Test– English language
Directions: (Q1 – Q10)Each of the blanks in each of the questions below can be filled by only two of the five words, other
of them making two sentences identical in meaning. Select the two options.

1. Despite their initial fears, most environmentalists now concede that the artificial reefs have had a largely _______
effect on surrounding ecosystems.
(A) unfounded (B) benign (C) caustic (D) interminable
(E) innocuous (F) plaintive

2. Scholarship reductions and player defections notwithstanding, the new coach applied himself to rebuilding the
program with such _______ that the rest of the staff struggled to match his enthusiasm.
(A) cessation (B) indifference (C) rhetoric (D) fervency
(E) heedlessness (F) zeal

3. After hours of practice and innumerable fruitless attempts to catch the balls, Allen was originally forced to admit
that he wasn’t sufficiently _______ to be a juggler.
(A) sedate (B) lumbering (C) dexterous (D) implicit
(E) adroit (F) awkward

4. The cohesion of Alexander the Great’s vast empire was _______ at his death, Alexander’s lands were divided
among his generals, Ptolemy, Seleucus, and Antigonus the One-Eyed.
(A) abiding (B) precarious (C) protracted (D) redoubled
(E) renowned (F) tenuous

5. His wife’s icy stare and aloof demeanor told Johann unequivocally that his propitiatory gifts had failed to _______
her anger.
(A) vilify (B) garner (C) exacerbate (D) aggravate
(E) placate (F) appease

6. By consuming _______ numbers of power bars, some athletes believe they will have proportionally greater amounts
of endurance and strength because of the energy-producing ingredients these products claim to contain.
(A) scant (B) furtive (C) copious (D) solvent
(E) measured (F) profuse

7. The comprehensive bill, signed into law by the president late last week, was _______ 249 new regulations on the
fishing industry.
(A) elucidated by (B) rife with (C) deficient in (D) unencumbered by
(E) replete with (F) exempted from

8. The former employees started a blog that revealed the embarrassing quirks of the boss, an act which had _______
impact on the company’s CEO.
(A) a virulent (B) an assuaging (C) a monumental (D) a discomfiting
(E) a bolstering (F) a mortifying

9. The late Samuel Huntington was well known for his _______ opinions on relations among different cultures; many
of his ideas are still passionately debated today.
(A) zealous (B) pedantic (C) polemical (D) rhetorical
(E) divisive (F) hegemonic

©LegalEdge Tutorials Page 31 of 60


Replication or other unauthorized use of this material is prohibited by the copyright laws of India
10. The _______ plant life on the previously barren volcanic rock created by the Kilauea lava Now is strong evidence
that humans, too, will one day be able to inhabit the area.
(A) incipient (B) nascent (C) waning (D) fervent
(E) flagging (F) static

Directions: Read the following paragraphs and carefully determine what the main idea is for each.
11. The best recipes for clam chowder all include onions and a bay leaf. The onions add a sharpness and zest to the
blandness of the clams, and also help remove their slimy texture. The bay leaf complements the onion’s strong
flavor.
This paragraph best supports the statement that___________
(a) onions were once thought to be poisonous.
(b) bay leaves are essential in many soups.
(c) clam chowder is very nutritious.
(d) onions and bay leaves go well with clams.
(e) clams should not be overcooked.

12. One New York publisher has estimated that 50,000 to 60,000 people in the United States want an anthology that
includes the complete works of William Shakespeare. And what accounts for this renewed interest in Shakespeare?
As scholars point out, the psychological insights he portrays in both male and female characters are amazing even
today.
This paragraph best supports the statement that___________
(a) Shakespeare’s characters are more interesting than functional characters today.
(b) people today are interested in Shakespeare’s work because of the characters.
(c) academic scholars are putting together an anthology of Shakespeare’s work.
(d) New Yorkers have a renewed interested in the work of Shakespeare.
(e) Shakespeare was a psychiatrist as well as a playwright.

13. Critical reading is a demanding process. To read critically, you must slow down your reading and, with pencil in
hand, perform specific operations on the text. Mark up the text with your reactions, conclusions, and questions.
When you read, become an active participant.
This paragraph best supports the statement that___________
(a) critical reading is a slow, dull, but essential process.
(b) the best critical reading happens at critical times in a person’s life.
(c) readers should get in the habit of questioning the truth of what they read.
(d) critical reading requires thoughtful and careful attention.
(e) critical reading should take place at the same time each day

14. There are no effective boundaries when it comes to pollutants. Studies have shown that toxic insecticides that have
been banned in many countries are riding the wind from countries where they remain legal. Compounds such as
DDT and toxaphene have been found in remote places like the Yukon and other Arctic regions.
This paragraph best supports the statement that___________
(a) toxic insecticides such as DDT have not been banned throughout the world.
(b) more pollutants fnd their way into polar climates than they do into warmer areas.
(c) studies have proven that many countries have ignored their own anti-pollution laws.
(d) DDT and toxaphene are the two most toxic insecticides in the world.
(e) even a worldwide ban on toxic insecticides would not stop the spread of DDT pollution.

©LegalEdge Tutorials Page 32 of 60


Replication or other unauthorized use of this material is prohibited by the copyright laws of India
15. The Fourth Amendment to the Constitution protects citizens against unreasonable searches and seizures. No search
of a person’s home or personal effects may be conducted without a written search warrant issued on probable
cause. This means that a neutral judge must approve the factual basis justifying a search before it can be conducted.
This paragraph best supports the statement that the police cannot search a person’s home or private papers unless
they have___________
(a) legal authorization.
(b) direct evidence of a crime.
(c) read the person his or her constitutional rights.
(d) a reasonable belief that a crime has occurred.
(e) requested that a judge be present.

Directions: answer the following multi choice questions based on the concepts of Grammar in usage.

16. A noun can be which of the following?


(a) The subject (b) An object (c) Predicative (d) A & B
(e) B & C (f) All of the above

17. What category of nouns is used to identify the person or group being directly spoken to?
(a) Common nouns (b) Nouns of address (c) Attributive nouns (d) Abstract nouns

18. Identify the type of noun (in bold) used in the following sentence: “Your indifference is not acceptable.”
(a) Proper noun (b) Countable noun (c) Collective noun (d) Abstract noun

19. What category of nouns is used to modify other nouns?


(a) Common nouns (b) Nouns of address (c) Attributive nouns (d) Abstract nouns

20. Which of the following is commonly used to create a noun from a verb or adjective?
(a) Prefix (b) Suffix (c) Attributive noun (d) Predicative noun

21. Which of the following cannot be used with countable nouns?


(a) Indefinite articles (b) Definite articles
(c) Third-person singular pronouns (d) Plural forms
(e) All of the above (f) None of the above

22. True or False: Countable nouns are always concrete nouns.


(a) True (b) False

23. How do most verbs conjugate when they have a singular subject?
(a) By adding “-d” to the end (b) By adding “-s” to the end
(c) They remain in their base form (d) They take an auxiliary verb

24. Which of the following third-person pronouns is used for plural nouns?
(a) he (b) she (c) it (d) they

25. Which of the following third-person pronouns is used for non-gendered singular nouns?
(a) he (b) she (c) it (d) they

26. Which of the following is a distinguishing feature of abstract nouns?


(a) They can be seen or touched (b) They cannot be seen or touched
(c) They can be counted (d) They cannot be counted

27. Proper nouns are generally _________.


(a) concrete (b) abstract

©LegalEdge Tutorials Page 33 of 60


Replication or other unauthorized use of this material is prohibited by the copyright laws of India
28. Is the following word concrete or abstract? “amazement”
(a) concrete (b) abstract

29. Is the following word concrete or abstract? “sugar”


(a) concrete (b) abstract

30. Is the following word concrete or abstract? “Australia”


(a) concrete (b) abstract

31. True or False: All concrete nouns are countable.


(a) True (b) False

Directions: (Q32-Q35)
The Renaissance, coined by Giorgio Vasari as the rebirth of art and literature by great men of Genius such as Michelangelo
and Niccolό Machiavelli, occurred in the fourteenth and fifteenth centuries in Italy (in all of Europe Italy had the most
impact). Aside from art and literature, the renaissance showed the changing in philosophy from everything being based on
religion to the idea of human nature and the creation of humanists. Major breakthroughs during the renaissance include
printing leading to the Gutenberg bible in 1456 and the political achievement of the residential ambassador. Humanism,
and humanists, was the evolution of thought from everything about God to the ideas and philosophies about human nature,
human beings, individual achievements, and the individual’s interests and capabilities. Humanists believed everyone
(mostly men), instead of being solely the wealthy and noble, should be educated and therefore schools were opened in most
cities. Some of the most famous humanists were Sir Thomas More and Desiderius Erasmus.

32. What is the appropriate title for the above passage?


(a) philosophy of the Renaissance
(b) Renaissance and God
(c) Humanity and Renaissance
(d) Changes during Renaissance

33. What according to the author is the meaning of 'Renaissance'?


(a) Rebirth of God (b) Complete Rebirth (c) Reinstated Philosophy (d) Humanistic changes

34. From the following changes, all are an outcome of the "Renaissance", except
(a) concepts on God and human nature
(b) printing of the Bible
(c) education of men
(d) political achievement of the local ambassador

35. The Renaissance was put together as the rebirth of art and literature by
(a) Sir Thomas More (b) Michelangelo and Niccol Machiavelli
(c) Giorgio Vasari (d) Desiderius Erasmus

Directions: (Q36-Q40)
Samina was very sad today because she had a Maths test on Wednesday and she had not studied in the two days that had
been given for preparations. She was lazy in everything and had a few friends only Another bad habit of hers was that she
didn't obey her parents, like yesterday when her mom told her to study but she stubbornly ignored her mother and went
to her room. In the morning, she woke up, put on her uniform, took her bag and went down where her breakfast was on
the table waiting for her. She said to her mother while taking a sip of tea, 'The tea is very hot and I am not drinking it/' She
gobbled down her breakfast and when she heard the horn of the van, she quickly went down. She jumped into her van and
sat down with her friends. In the class, the first period was of Urdu and she did not focus on anything. When the teacher
gave them some class work, she kept sitting there blankly and asking from other children. "Samina have you studied for
the maths test?" her friend Maliha asked her. "Not yet, I have one more day left. The test is tomorrow." Samina said but in
her heart, she was saying that I will not study today either. At home she asked her mother to give her some food because
she was very tired. When her mother did so, she said, "I will not eat this. I hate this food." She threw the plate on the floor
and started to play on her tablet while her mother cleaned the food from the carpet. The next day was the same. She failed

©LegalEdge Tutorials Page 34 of 60


Replication or other unauthorized use of this material is prohibited by the copyright laws of India
in the Maths test and gave an excuse to her teacher. Every day the same routine continued till the day of her exam came
near but she was not worried. She played on the tab at home all the time and did not study at all.
At last it was the day of her exams and she was thinking how to pass but she could not. Now she was worried. In the paper
she was able to answer only some of the questions and left most of the others unanswered. In all the exams, the same thing
happened and the result was as expected.
On the result day, she stayed at home and only her mother went there and when she came home she was very angry as
Samina had failed very badly. Her mother took her tab and mobile away and did not talk to her for many days. Samina
apologised but her mother did not accept it. So Samina started to study and at last her mother started to talk to her. After
that day she was an obedient child and became very good in studies. Her mom loved her too.
36. The probable reason why Samina failed could be any of the following, except
(a) her indolence (b) her adolescence (c) her ignorance (d) her obstinacy

37. Which of the following is true as per the passage?


(a) Samina apologised as she wanted her tab and mobile back.
(b) Sam inn was doing all this for her mother's attention,
(c) Samina felt guilty of her behavior and carelessness.
(d) Saminn was in bad company due to which she failed.

38. The above extract is probably a


(a) Fairy-tale (b) true story (c) moral story (d) work of fiction

39. The above story is a lesson in terms of


(a) Parents need to be more attentive to their children.
(b) Company really matters to build or destroy a child’s caliber.
(c) Students need to be obedient only then can they pass.
(d) Realizing ones mistakes and not repeating them can be fruitful.

40. Fill in the blanks with the correct alternative Caw is to crows as ____ is to cows.
(a) bleat (b) snort (c) low (d) bellow

©LegalEdge Tutorials Page 35 of 60


Replication or other unauthorized use of this material is prohibited by the copyright laws of India
Current Affairs Practice Sheet (CAPS)

1. _________ has topped the list of states with the (c) third (d) first
highest employability as per the India Skills
Report 2019. 10. Who has launched the Institution’s Innovation
(a) Gujarat (b) Andhra Pradesh Council through video conferencing in New Delhi
(c) Madhya Pradesh (d) Assam recently?
(a) Prakash Javadekar (b) Sushma Swaraj
2. _______has recently partnered with the Life (c) Rajnath Singh (d) Narendra Modi
Insurance Corporation of India (LIC) to offer
online insurance premium payments on its 11. Laos is a Southeast ____________ country.
platform. (a) European (b) American
(a) Paytm (b) Google Pay (c) Asian (d) African
(c) True Caller (d) WhatsApp
12. The _____________ ‘UK India Business Council
3. 7th International Tourism Mart for the North-East Ease of Doing Business Report’ has been released.
States held in which of the following states? (a) fifth (b) fourth
(a) Assam (b) Tripura (c) third (d) second
(c) Meghalaya (d) Manipur
13. The World Health Organisation (WHO) in a report
4. Which of the following country has got UN stated that ______________ is the only country to
Environment Award for combating have recorded a substantial decline in malaria
Transboundary Environmental Crime? cases in 2017 out of the 11 highest burden
(a) Norway (b) Bhutan countries worldwide.
(c) Nepal (d) India (a) India (b) China
(c) France (d) USA
5. Who has won Forbes India Leadership Awards
2018, Lifetime Achievement? 14. Which organisation has won this year’s Indira
(a) Azim Premji (b) Mukesh Ambani Gandhi Prize for Peace, Disarmament and
(c) Shantanu Narayan (d) N R Narayan Murthi Development, an award conferred by Indira
Gandhi Trust each year on the late Prime
6. Prime Minister Shri Narendra Modi laid the Minister’s birth anniversary?
Foundation Stones of _________Projects in 65 (a) Council of Scientific & Industrial Research
Geographical Areas (GAs) in 129 Districts. (b) University Grants Commission
(a) Clean Ganga (c) Centre for Science and Environment
(b) Agri-loan Distribution (d) Indian Oil Corporation
(c) Power Line Connectivity
(d) City Gas Distribution 15. _______________________ has inked an agreement
with the US Trade and Development Agency
7. The second edition of the India-UAE Strategic (USTDA) to develop a roadmap for modernization
Conclave will be held in which of the following of its air traffic services.
city? (a) Air India
(a) Hyderabad (b) Dubai (b) Research and Analysis Wing
(c) Abu Dhabi (d) New Delhi (c) Ministry of Road Transport and Highways
(d) Airports Authority of India
8. Who is the present Petroleum Minister of India?
(a) V K Singh 16. Who has Sworn in as Fiji’s Prime Minister for four
(b) Jitendra Kumar more years after winning an election with a
(c) Dharmendra Pradhan reduced majority?
(d) Harsh Vardhan (a) Thomas Kurian
(b) Michael D Higgins
9. President of India Ram Nath Kovind was in (c) Voreqe Bainimarama
Australia on his second leg of 2-Nation visit. He (d) Jair Bolsonaro
became the ____________ Indian President to visit
Australia.
(a) fifth (b) fourth

©LegalEdge Tutorials Page 36 of 60


Replication or other unauthorized use of this material is prohibited by the copyright laws of India
17. Centre for Science and Environment is a not-for-
profit public interest research and advocacy 26. Who among the following has been named by
organisation based in- Fortune in its 2018 Business Person of the Year
(a) Chennai (b) New Delhi list?
(c) Kolkata (d) Mumbai (a) Sundar Pichai (b) Satya Nadella
(c) Shantanu Narayen (d) Sabeer Bhatia
18. The United Nations children’s agency UNICEF
appointed _______________________ as its 27. ‘Radio Kashmir-In Times of Peace & War’ is
youngest ever goodwill ambassador recently. authored by _____________.
(a) Millie Bobby Brown (a) Rajesh Bhat (b) Kiran Desai
(b) Benjamin Bratt (c) Vikram Seth (d) Salman Rushdie
(c) Ryan Destiny
(d) Naomi Campbell 28. National Stock Exchange of India (NSE) has
launched a mobile application and web-based
19. The International Cricket Council (ICC) platform for retail investors to buy government
announced its partnership with securities. What is the name of this app?
_________________ to offer the fans a great (a) WORTH to BID (b) NSE goCORE
viewing experience of the upcoming semi-finals (c) NSE WORTH (d) NSE goBID
and final of the ICC Women’s World T20 to be
held in West Indies. 29. Which of the following country has topped the list
(a) Google (b) Microsoft of the IMD World Talent Ranking 2018 for the fifth
(c) Facebook (d) IBM year in a row?
(a) Norway (b) Finland
20. Who has been elected as Interpol’s next president? (c) Switzerland (d) Singapore
(a) Alexander Prokopchuk
(b) Meng Hongwei 30. United Nations Universal Children’s Day is
(c) Jurgen Stock celebrated on ________.
(d) Kim Jong-yang (a) November 22nd (b) October 2nd
(c) November 14 th (d) November 20th
21. The International Criminal Police Organization,
more commonly known as Interpol, is the 31. Which among the following country won the 59th
international organization that facilitates edition of the junior boys Subroto Cup
international police cooperation. Where is the International Football Tournament?
headquarters of Interpol? (a) Afghanistan (b) Bangladesh
(a) Canada (b) France (c) Nepal (d) India
(c) Russia (d) USA
22. Joint military exercise ‘Vajra Prahar’ recently 32. The ________ cabinet has approved to rejoin the
commenced in ______. Commonwealth.
(a) Uttarakhand (b) Jodhpur (a) Australia (b) Sri Lanka
(c) Bhopal (d) Jaipur (c) Scotland (d) Maldives

23. Vajra Prahar, a Joint military exercise took place 33. India’s first specialized hospital for elephants has
between India and _______. been opened at_________.
(a) Russia (b) United States (a) Agra (b) Mathura
(c) South Korea (d) China (c) Kanpur (d) Bhopal

24. India’s first ‘Justice City’ will be set up within 34. The Asia-Pacific Economic Cooperation (APEC)
which of the following city? summit was recently held in________.
(a) Ahmadabad (b) Hyderabad (a) France (b) Spain
(c) Amaravati (d) Jaipur (c) Brazil (d) Papua New Guinean

25. The 27th BASIC Ministerial Meeting on Climate 35. The 14th edition of JashneBachpan, a theatre
Change was held in _______. festival for children, held at _______.
(a) Rio de Janeiro (b) Shanghai (a) New Delhi (b) Bhopal
(c) Beijing (d) New Delhi (c) Gwalior (d) Ayodhya

©LegalEdge Tutorials Page 37 of 60


Replication or other unauthorized use of this material is prohibited by the copyright laws of India
36. Lakshya Sen won bronze medal at Li Ning BWF 45. ‘Battle of Aberdeen’ is associated with
World Junior Badminton Championships 2018. It (a) Natives of the Andaman Islands
was held in _____. (b) Natives of the Lakshadweep Islands
(a) Japan (b) Canada (c) Rohingyas of Myanmar
(c) The USA (d) China (d) Matuas of Bengal

37. Africa Industrialization Day (AID) is celebrated on 46. ‘Virtual Climate Summit’, is the brainchild of
__________. (a) Denmark (b) Sweden
(a) 23 November (b) 11 November (c) Norway (d) Marshall Islands
(c) 16 November (d) 20 November
47. Consider the following statements with respect to
38. The 49th edition of the International Film Festival ‘Climate Vulnerable Forum (CVF)’
of India held in ________. 1. It was formed to increase the accountability of
(a) Mumbai (b) Jharkhand industrialized nations for the consequences of
(c) Goa (d) Paris global climate change.
2. It was founded by the Maldives during the 2015
39. Which of the following country is the focus United Nations Climate Change Conference
country of 49th edition of the International Film Select the correct statements
Festival? (a) 1 Only (b) 2 Only
(a) France (b) Israel (c) Both 1 and 2 (d) Neither 1 nor 2
(c) Australia (d) Nepal
48. Consider the following statements
40. The World Toilet Day is organized around the 1. The epistemology of Vaiśeṣika school of
world on ______. Hinduism accepted only two reliable means to
(a) 24th November (b) 17th November knowledge - perception and inference
(c) 21th November (d) 19th November 2. Samkhya is the ancient school of Indian
materialism
41. Consider the following statements with respect to Select the correct statements
‘Hansen’s disease’ (a) 1 Only (b) 2 Only
1. It is a bacterial disease (c) Both 1 and 2 (d) Neither 1 nor 2
2. It affects the skin and nerves which can lead to
physical deformity and disability if left untreated 49. ‘Green Growth Equity Fund’ is a joint fund
Select the correct statements between
(a) 1 Only (b) 2 Only (a) BRICS Countries
(c) Both 1 and 2 (d) Neither 1 nor 2 (b) Quad Countries
(c) Member of International Solar Alliance
42. Consider the following statements with respect to (d) None of the above
‘Mountain gorilla’
1. It is native to South East Asia 50. Consider the following statements about
2. It is listed as critically endangered under the Permanent Structured Cooperation (PESCO)
IUCN Red list 1. It is an intergovernmental, binding, permanent
Select the correct code: framework and a structured process to
(a) 1 Only (b) 2 Only gradually deepen defence cooperation within
(c) Both 1 and 2 (d) Neither 1 nor 2 European Union framework
2. It is formed to counter the North Atlantic Treaty
43. ‘Rothschild’s giraffe’ is listed as ________ under Organization
IUCN Red List Select the correct statements
(a) Endangered (b) Near Threatened (a) 1 Only (b) 2 Only
(c) Vulnerable (d) Extinct (c) Both 1 and 2 (d) Neither 1 nor 2

44. The place called ‘Lusaka’ was recently in news. 51. 'Project C.A.T’ is
Where is it located? (a) A trap-neuter-return initiative for community
(a) South East Asia (b) Central Asia cats in Mumbai
(c) Northern Africa (d) None of the above

©LegalEdge Tutorials Page 38 of 60


Replication or other unauthorized use of this material is prohibited by the copyright laws of India
(b) A project aimed at conserving the wild tiger
population, by Discovery Communications 58. Which team won the 'ICC Women's World T20
and World Wildlife Fund (WWF) title' for the fourth time?
(c) A concept for a unmanned single-stage (a) West Indies (b) India
reusable spaceplane capable of horizontal (c) Australia (d) England
take-off and landing by HAL
(d) None of the above 59. Which country to Lay Foundation Stone For
Kartarpur Corridor Route?
52. ‘Aghanashini Lion Tailed Macaque Conservation (a) India (b) Pakistan
Reserve’ is located in (c) Saudi Arabia (d) Indonesia
(a) Kerala (b) Karnataka
(c) Tamil Nadu (d) Goa 60. Which country has set a Guinness world record
with plankathon?
53. Consider the following statements with respect to (a) Japan (b) Russia
‘Ease of Doing Business Grand Challenge’ (c) SriLanka (d) India
1. The objective of this challenge is to invite
innovative ideas based on cutting edge technology 61. According to Organisation for economic
to reform Government processes. Cooperation and Development(OECD), India’s
2. The platform for the Grand Challenge is the GDP to remain at how much percent in 2019
Startup India Portal. and 2020?
Select the correct statements (a) 7.5 percent (b) 7.3 percent
(a) 1 Only (b) 2 Only (c) 7.2 percent (d) 7.8 percent
(c) Both 1 and 2 (d) Neither 1 nor 2
62. Which country bagged their 4th Women’s
54. Consider the following statements with respect to World T20 crown 2018?
‘Capital Adequacy ratio (CAR)’ (a) Pakistan (b) England
1. It measures a bank's capital in relation to its (c) Australia (d) India
risk-weighted assets.
2. CAR of 9% is prescribed under the Basel III 63. Who won the gold medal in the Sabre event in
norms the Senior Commonwealth Fencing
Select the correct statements Championship 2018 held in Canberra,
(a) 1 Only (b) 2 Only Australia?
(c) Both 1 and 2 (d) Neither 1 nor 2 (a) Husanpreet Kaur (b) Bhupen Singh
(c) Karan Singh (d) Bhavani Devi
55. The term ‘Economic Capital Framework (ECF)’
was in news recently in the context of 64. Which Indian player registered history in the
(a) Reserve Bank of India (RBI) T10 cricket league, where he stunned Kerala
(b) World Trade Organisation (WTO) Knights with a five-wicket haul, including a hat
(c) Ease of Doing Business trick?
(d) MSMEs (a) Himanshu Rana (b) Harvik Desai
(c) Manjot Kalra (d) Pravin Tambe
56. Which of the following Indian cities have been
selected by UN to participate in Global 65. Recently, Ricky Jay passed away. He was
Sustainable Cities 2025 initiative? related to which field?
(a) Ahmedabad, Lucknow (a) Musician (b) Actor
(b) Surat, Pune (c) Philanthropist (d) Magician
(c) Noida, Greater Noida
(d) Bhopal, Visakhapatnam 66. What was the theme of this year’s International
Day for the Elimination of Violence Against
57. The two Indian movies competing for the Women?
UNESCO Gandhi Medal are ________. (a) Orange The World: #HearMeToo
(a) To Let and Paddayi (b) Say No:Unite to End Violence
(b) Peranbu and Olu (c) Take Action: Say No To Violence
(c) Pariyerum Perumal and Mahanati (d) Leave No One Behind: End Violence
(d) Baaram and Walking with the Wind Against Women and Girls

©LegalEdge Tutorials Page 39 of 60


Replication or other unauthorized use of this material is prohibited by the copyright laws of India
77. 12th World Congress on mountain Medicine
67. Which country’s PM arrived India with the aim started in which city?
to repair the bilateral ties which came under (a) Beijing (b) New Delhi
severe strain during the previous government (c) Tokyo (d) Kathmandu
in the island nation?
(a) Netherlands (b) Australia 78. Who was appointed as UNICEF’s youngest ever
(c) Maldives (d) Austria goodwill ambassador?
(a) Roger Moore
68. Baliyatra Festival was inaugurated in which (b) Millie Bobby Brown
city? (c) Richard Attenboro
(a) Dispur (b) Pune (d) Leon Lai
(c) Gnadhinagar (d) Cuttack
79. When is World Hello day observed?
69. Which Indian boxer has created history by (a) Nov 22 (b) Nov 15
becoming India’s only six time world boxing (c) Nov 21 (d) Nov 20
champion?
(a) Mary Kom (b) Pinki Rani 80. The 27th BASIC Ministerial Meeting on Climate
(c) Kavita Goyat (d) Sarjubala Devi Change was held in which city?
(a) Lucknow (b) Pune
70. Men’s Hockey World Cup Anthem which is to (c) Mumbai (d) New Delhi
be held in Odisha is composed by whom?
(a) Anu Malik (b) Amit Trivedi 81. What is the rank of India in IMD World Talent
(c) Pritam (d) A. R Rahman Ranking 2018?
(a) 53rd (b) 50th
71. Didier Drogba has confirmed his retirement as a (c) 55th (d) 52nd
player.He was related to which sports?
(a) Cricket (b) Football 82. Prime Minister Narendra Modi laid the
(c) Baseball (d) Badminton foundation stone of Vishwakarma Skill
University in which city?
72. The 50th Union World Conference on Lung (a) Kanpur (b) Lucknow
Health will be held in which city? (c) Nashik (d) Gurugram
(a) Gangtok (b) Ahmedabad
(c) Hyderabad (d) Pune 83. Which application was launched by Google
which is specifically designed for the Indian
73. Sangai festival 2018 begins in which state? market, will give hyper local information on
(a) Sikkim (b) Assam neighbourhoods?
(c) Mizoram (d) Manipur (a) NeighbourHood Me
(b) Locals
74. Which carpet city got the major boost as the (c) Neighbour
government has extended the “export (d) Neighbourly
excellence tag”?
(a) Kolhapur (b) Solapur 84. Which country won 59th edition of Subroto
(c) Bhadohi (d) Gorakhpur Cup International Football Torurnament?
(a) Bangladesh (b) India
75. According to LinkedIn, which city pays the (c) Maldives (d) China
highest salary in India?
(a) Bengaluru (b) Mumbai 85. 87th session of the INTERPOL General
(c) Delhi (d) Chennai Assembly held in which city?
(a) Paris (b) Dubai
76. President Ramnath Kovind unveiled a statue of (c) New York (d) Abu Dhabi
Mahatma Gamdhi in which city?
(a) Melbourne (b) Sydney 86. When is World Television Day observed?
(c) New York (d) Paris (a) Dec 12 (b) Nov 21
(c) Nov 20 (d) Oct 21

©LegalEdge Tutorials Page 40 of 60


Replication or other unauthorized use of this material is prohibited by the copyright laws of India
87. The 14th edition of international children’s (a) MeBID (b) GiveBid
theatre festival, “Jashne Bachpan”, was (c) goBID (d) BIDs
inaugurated in which city?
(a) Pune (b) New Delhi 96. Which country has decided to to rejoin the
(c) Lucknow (d) Mumbai Commenwealth, two years after withdrawing
from the 53-nation grouping?
88. RBI Central Board decided to constitute a BFS (a) Maldives (b) Mauritius
of the Central bank to examine the framework (c) SriLanka (d) Pakistan
for banks under PCA.What is “S” in BFS?
(a) Super (b) Supervision 97. Who was sworn as new Fiji PM?
(c) Syndrome (d) System (a) Frank Bainimarama
(b) Faiyaz Koya
89. The 12-day long joint military exercise between (c) Jioji Konrote
India and the USA known as “Vajra (d) Voreque Bainimarama
Prahar”commenced in which city?
(a) Jaipur (b) Mumbai 98. Who was awarded this year’s Indira Gandhi
(c) Bengaluru (d) Chennai Prize for Peace,Disarmament and
Development?
90. According to a WHO report, which is the only (a) Centre for science and Environment
country to have recorded a substantial decline (b) NABARD
in malaria cases in 2017 by 24 percent? (c) IISR
(a) SriLanka (b) Australia (d) AIIMS
(c) India (d) Nepal 99. What is the theme of this year’s United Nations’
(UN) Universal Children’s Day?
91. Prime Minister Narendra Modi constituted a (a) Improving Child Welfare Worldwide
National Implementation Committee(NIC) (b) KidsTakeOver
under the chairmanship of which minister? (c) Stop Violence against Children
(a) Arvind Subramaniyam (d) Children are taking over and turning the
(b) Rajnath Singh world blue
(c) Smriti Irani
(d) Arun Jaitley 100. Which organization launched its e-commerce
portal recently?
92. Which ministry has introduced the upgraded (a) NICL (b) India Post
version of AirSewa 2.0 web portal? (c) SIDBI (d) NABARD
(a) Ministry of Civil Aviation
(b) Ministry of Defence 101. The two-day summit to mark the World
(c) Ministry of Finance Tolerance Day was inaugurated in which city?
(d) Ministry of Human Resources Development (a) Dubai (b) Paris
(c) Bishkek (d) Kiev
93. The world’s standard definitions of the
kilogram,the ampere,the Kelvin and the mole 102. Which word was declared as the word of the
has been changed after how many years? year by Oxford Dictionary?
(a) 130 years (b) 135 years (a) Gaslighting (b) Poisonous
(c) 140 years (d) 150 years (c) Toxic (d) Incel

94. Who is the author of “The Velvet Gloves”? 103. Credit rating agency Fitch affirmed which
(a) Sudeep Nagarkar rating to India’s “Long Term Foreign Currency
(b) Balakrishna Kamath Issuer Default Rating(IDR)”?
(c) Bala Sagar Singh (a) AA- (b) A+
(d) Ravi Subramanaian (c) BB+ (d) BBB-

95. Which application was launched by National 104. Which company acquired Artificial Intelligence
Stock Exchange of India for retail investors to (AI) and bot development company XOXCO for
buy government securities? an undisclosed sum.?

©LegalEdge Tutorials Page 41 of 60


Replication or other unauthorized use of this material is prohibited by the copyright laws of India
(a) Accenture (b) Microsoft
(c) Google (d) TCS 115. Which state to provide free education to Ockhi
storm- hit kids?
105. What is the theme of this year’s World (a) Tamil Nadu (b) Kerala
Tolerance Day 2018? (c) Telangana (d) Odisha
(a) Prospering from Pluralism: Embracing
Diversity through Innovation and 116. Union cabinet recently approved the strategic
Collaboration. sale of 100 percent government stake in which
(b) Lets nurture tolerance and respect diversity organization?
(c) Fighting tolerance (a) BEL
(d) Together (b) BHEL
(c) IDBI Bank
106. When is National Press Day celebrated? (d) Dredging Corporation of India
(a) Nov 16 (b) Nov 20
(c) Nov 10 (d) Nov 17 117. Which Indian wrestler touched a new high in
his career by achieving the number one rank in
107. China tapped which country as digital economy the world in the 65kg category?
partner? (a) Pawan Kumar (b) Bajrang Punia
(a) Sri Lanka (b) Nepal (c) Yogeshwar Dutt (d) Sushil Kumar
(c) Pakistan (d) India
118. The 34th National Junior Athletic
108. The election commission has set which polling Championship was held at which place?
booths in various constituencies of Chattisgarh (a) Shimla (b) Gangtok
to encourage women voters to exercise their (c) Ranchi (d) Lucknow
franchise in the Assembly elections?
(a) Samridhi (b) Sarojana 119. Which project has won UNESCO Asia Pacific
(c) Sangwari (d) Saregama award for conservation?
(a) Jammu Project (b) Sikkim Project
109. The Supreme Court has given nod for a (c) Ladakh Project (d) Leh Project
separate High Court for which state?
(a) Sikkim (b) Odisha 120. Who was awarded the reporters without
(c) Uttar Pradesh (d) Andhra Pradesh Borders(RSF) Press Freedom Award for
Courage?
110. Which country will hold the 5th World Internet (a) Anvesha Sharma
Conference? (b) Arundhati roy
(a) India (b) China (c) Minisha Singh
(c) Russia (d) Japan (d) Swati Chaturvedi
111. Which country has unveiled world’s first AI
news anchor? 121. Which actress has unveiled her book “Healed”,
(a) USA (b) Russia her personal story of a battle against ovarian
(c) Japan (d) China cancer?
(a) Manisha Koirala
112. Which became the first country in the world to (b) Katrina Kaif
ban reef killing sunscreen? (c) Tabu
(a) Andorra (b) Albania (d) Rekha
(c) Palau (d) Uganda
122. The 25th edition of the Singapore India
113. When is National Education Day celebrated? Maritime Bilateral Exercise(SIMBEX) kicked off
(a) Nov 7 (b) Nov 9 at which place?
(c) Nov 11 (d) Nov 10
(a) Vijaywada (b) Gandhinagar
(c) Port Blair (d) Vadodara
114. Vice President Venkaiah Naidu inaugurated
Indian War Memorial in which country? 123. The Cabinet approved setting up of a Central
(a) Australia (b) Russia
Tribal University in which state?
(c) China (d) France

©LegalEdge Tutorials Page 42 of 60


Replication or other unauthorized use of this material is prohibited by the copyright laws of India
(a) Uttar Pradesh (b) Andhra Pradesh (a) Pakistan (b) Ethiopia
(c) Bihar (d) Odisha (c) Saudi Arabia (d) North Korea

124. Which body is likely to establish an 134. Who Won Bronze Medal In World Wrestling
ombudsman for digital payments by March Championships?
2019? (a) Pooja Dhanda (b)Varun Karthik
(a) IRDA (b) RBI (c) Dinesh Gupta (d) Suresh Kumar
(c) NABARD (d) SIDBI
135. Which country Becomes Second Largest Solar
125. Which Indian players won gold at the 10m Air Market: MERCOM?
Pistol mixed team event on the last day of the (a) India (b) Pakistan
Asian Airgun Chanpionship in Kuwait? (c) Saudi Arabia (d) Indonesia
(a) Mohd Asab and Suma Shirur
136. Which place NATO's Biggest Military Maneuvers
(b) Ankur Mittal and Avneet Sidhu
(c) Saurabh Chaudhary and Manu Bhaker Begins ------- ?
(a) Norway (b) China
(d) Karam Lehal and Saniya Sheikh
(c) Australia (d) Denmark
126. Which Indian women’s player became the first
Indian to score a century in T20 Internationals? 137. Where is the headquarters of United Nations?
(a) New york (b) Russia
(a) Sushma Verma
(b) Smriti Mandhana (c) Geneva (d) Denmark
(c) Mithali Raj
(d) Harmanpreet Kaur 138. Which of the following did Madras HC ban
regarding drugs?
127. Which batsman became the first batsman to hit (a) False practising
four centuries in T20 internationals? (b) Selling for high price
(a) Rohit Sharma (b) Darren Bravo (c) Online drug sale
(c) Virat Kohli (d) Faf Du Plessis (d) drug that are banned by USFDA
128. India’s biggest social media summit,”Social 139. Vice President M Venkaiah Naidu on 31 October
Media Summit and Awards 2018”kicked off in 2018 embarked on a three-nation visit to
which city? Botswana, Zimbabwe and ____________
(a) Kolkata (b) Vijaywada (a) Ethiopia (b) Tanzania
(c) Lucknow (d) NewDelhi (c) Nigeria (d) Malawi
129. Who will be awarded with “Global Education 140. Which country has recently announced to
Award”on November 8 by “Malabar Gold and withdraw from United Nations' (UN) migration
Diamonds”? pact?
(a) Binny Bansal (b) Anand Kumar (a) Brazil (b) Australia
(c) Subash Chandra (d) Satish Kaushik (c) Canada (d) France

130. When is National Cancer Awareness Day 141. Which naval ship returned after 205-day world
observed? tour?
(a) Nov 7 (b) Nov 8 (a) INS Tarangini (b) INS Viraat
(c) Nov 9 (d) Nov 10 (c) INS Satpura (d) INS Shakthi

131. Which state has decided to observe “Rosogolla 142. Under which department did Tamil Nadu govt.
Day” on November 14? provide assistance for women for their
(a) Bihar (b) Maharashtra emergency?
(c) West Bengal (d) Odisha (a) Department of Women and Child welfare
(b) Department of Social Welfare and Nutritious
132. Which company fired 48 employees for sexual Meal Programme
harassment in past two years? (c) Department of Human Affairs
(a) Google (b) Samsung (d) Department of Human Welfare and Nutritious
(c) Apple (d) Verizon Meal Programme
133. Which country appoints Sahle-Work Zewde As 1st 143. Which rank did India obtained in Global Passport
Female President?
Index?

©LegalEdge Tutorials Page 43 of 60


Replication or other unauthorized use of this material is prohibited by the copyright laws of India
(a) 65 (b) 25 154. Which of the following country unveiled a
(c) 35 (d) 28 sculpture in honour of Indian soldiers who fought
during World War I?
144. Which rank did India secure in the World Bank’s (a) US (b) UK
Ease of Doing Business Index 2018? (c) German (d) France
(a) 103rd (b) 77th
(c) 54th (d) 89th 155. World Tsunami Awareness Day is being observed
on _______.
145. Which country inked a $950 million deal for (a) November 5 (b) November 7
inducting two new warships equipped with (c) November 4 (d) November 6
Brahmos missiles in the India Navy?
(a) Russia (b) Indonesia 156. Delhi's Signature Bridge is built over the ______
(c) Saudi Arabia (d) Pakistan river.
(a) Sahibi (b) Ganga
146. 49th International Film Festival of India, IFFI 2018, (c) Brahmaputra (d) Yamuna
will be held in Goa. Olu, the opening film in IFFI
2018 is made in which of the following language? 157. Which country will host a Formula One race in
(a) Tulu (b) Telugu April 2020?
(c) Malayalam (d) Tamil (a) Vietnam (b) Malaysia
(c) Germany (d) Philippines
147. Who replaces Elon Musk as Tesla's Chairman with
immediate effect? 158. Who became the fifth Indian to be inducted in the
(a) Irene Rosenfeld (b) Sheryl Sandberg 'ICC Hall of Fame'?
(c) Robyn Denholm (d) Marissa Mayer (a) Anil Kumble (b) Sunil Gavaskar
148. _______ launches fake news fight-back with global (c) Rahul Dravid (d) Kapil Dev
campaign?
(a) BBC (b) NBC 159. The National Cancer Awareness Day is observed
(c) ITBC (d) NDTV on which date in India?
(a) November 7 (b) November 8
149. Trojan asteroids are associated with _____ planet. (c) November 5 (d) November 6
(a) Venus (b) Mercury
(c) Mars (d) Jupiter 160. Which Cricket Stadium has been recently renamed
after Atal Bihari Vajpayee, a day before India-WI
150. India lost a major trade dispute over safeguard T20I?
duty on hot-rolled steel at the World Trade (a) Trivandrum’s Greenfield International
Organization (WTO). The case was filed by Stadium
________ in December 2017. (b) Bangalore’s M. Chinnaswamy Stadium
(a) China (b) Japan (c) Lucknow's Ekana International Cricket
(c) Germany (d) Australia Stadium
(d) Ranchi’s JSCA International Cricket Stadium
151. Which country unveils ‘most advanced’ stealth
drone - UAV – CH7? 161. Who became first cricketer in history to score 4
(a) China (b) South Africa T20I hundreds?
(c) Pakistan (d) Sri Lanka (a) Steven Smith (b) David Warner
152. India's first nuclear-powered submarine _______ (c) Rohit Sharma (d) Virat Kohli
has completed its first deterrence patrol recently.
162. Recently, Supreme Court has given nod for a
(a) INS Sahyadri (b) INS Kalvari
(c) INS Chakra (d) INS Arihant separate High Court for Andhra Pradesh. The new
court will be the ______th High Court in the
country.
153. Where is the world’s largest marine sanctuary
planned to be built? (a) 25 (b) 26
(c) 34 (d) 30
(a) Arctic (b) Iceland
(c) Antarctica (d) Cyprus 163. Which country unveiled new 'Heavenly Palace'
space station, which is set to become the

©LegalEdge Tutorials Page 44 of 60


Replication or other unauthorized use of this material is prohibited by the copyright laws of India
replacement of the International Space Station (c) Egypt President Abdel Fattah el-Sisi
(ISS), which is due to retire in 2024? (d) South Africa president Cyril Ramaphosa
(a) Japan (b) United States
(c) Russia (d) China 173. Who is appointed as UNICEF India’s Youth
Ambassador?
164. Who is chosen for Raja Ram Mohan Roy Award (a) Hima Das (b) Siddharth Pardeshi
(c) Srihari Nataraj (d) Avinash Mani
by Press Council of India (PCI)?
(a) N Ram (b) Dinesh Karthik 174. Rs 75 coin to be issued by government to mark
(c) Suresh Rajan (d) Ganesh Kumar 75th anniversary of Tricolour hoisting by -------- ?
(a) Sardar Vallabhai Patel
165. At the Armistice of World War I centenary (b) Netaji Subhash Chandra Bose
commemoration, Vice President M Venkaiah (c) Jawaharlal Nehru
Naidu inaugurated the Indian Armed Forces (d) D.Mahatma Gandhi
Memorial in ________.
(a) Russia (b) Germany 175. Which communication satellite did ISRO recently
(c) France (d) Turkey launch on board its second heaviest
developmental flight GSLV-MkIII D2?
166. Armistice Day is commemorated on _______ to (a) GSAT-29 (b) GSAT-39
mark the end of the 1914-1918 conflict (World War (c) GSAT-49 (d) GSAT-69
I). 176. What is the name of the e-learning portal launched
(a) November 12 (b) November 14
by Delhi Police Commissioner Amulya Patnaik on
(c) November 13 (d) November 11 November 14, 2018?
167. Which of the following Indian Wrestler becomes (a) SWAYAM (b) DAKSH
World No.1 in 65 kg category? (c) NIPUN (d) STRIKE
(a) Bajrang Punia (b) Vinesh Phogat 177. Which country has initiated to create an
(c) Sandeep Tomar (d) Sakshi Malik international research center to fight against
resistance to antibiotics?
168. India and France signed a deal worth Euro 7.87 (a) Germany (b) France
billion (Rs 59,000 crore approximately) on (c) Denmark (d) India
September 23, 2016 for _______ Rafale jets.
(a) 34 (b) 32 178. Astronomers recently found a 'cold super-Earth'.
(c) 36 (d) 38 The planet has been labeled ___________.
(a) Barnard's Star e (b) Barnard's Star g
169. Prime Minister Narendra Modi inaugurated (c) Barnard's Star b (d) Barnard's Star d
India's first Multi-Modal Terminal on ______ river.
(a) Ganga (b) Brahmaputra 179. Where did the 62nd National shooting
(c) Godavari (d) Krishna championships, rifle, and pistol, begin?
(a) Kerala (b) Tamil Nadu
170. Name the Bangladesh cricketer, who will contest (c) Karnataka (d) Uttar Pradesh
in the upcoming elections on a ticket from the
ruling party, Awami League, as confirmed by PM 180. PM Narendra Modi launched the Government’s
Sheikh Hasina. Support and Outreach Initiative for Micro,Small
(a) Tamim Iqbal Khan and Medium Enterprises(MSME) in which city?
(b) Shakib Al Hasan (a) Mumbai (b) New Delhi
(c) Mashrafe Mortaza (c) Lucknow (d) Gorakhpur
(d) Mohammad Mithun

171. India celebrated _______th birth anniversary of Pt 181. Which decommissioned carrier will be converted
Jawaharlal Nehru on 14th November 2018? into India’s first ever moored maritime museum-
(a) 130 (b) 134 cum-marine adventure centre?
(c) 135 (d) 139 (a) INS Mhadei (b) INS Makar
(c) INS Tarini (d) INS Viraat
172. Who will be India's Republic Day chief guest in
2019? 182. Indian Air Force is organising a two day seminar
(a) Sudan President Omar al-Bashir on “Emerging Trends in aviation Engineering and
(b) Mauritius President Ameenah Gurib

©LegalEdge Tutorials Page 45 of 60


Replication or other unauthorized use of this material is prohibited by the copyright laws of India
Logistics(AVIAMAT-2018) in which city? 192. Recently which personality died who commanded
(a) Bhopal (b) NewDelhi a naval unit of the Eastern Fleet in the 1971 India
(c) Gwalior (d) Mumbai Pakistan War and whose actions led to the
destruction of an enemy submarine?
183. India signed agreement with which country for
(a) RP Hari (b) Ashok Kumar
the construction of the Turga Pumped storage (c) Ajit Kumar (d) MP Awati
hydel power project at Purulia in West Bengal?
(a) China (b) Japan 193. South Korean first lady Kim Jung Sook and Chief
(c) Russia (d) France
Minister Yogi Adityanath inaugurated the Queen
184. Which country will host the 23rd edition of the Huh Memorial in which city?
World Corporate Games? (a) Shirdi (b) Varanasi
(a) Afghanistan (b) Qatar
(c) Puri (d) Ayodhya
(c) Australia (d) USA
185. Which country will host Formula One Race in 194. Which district will be renamed as Ayodhya?
April 2020? (a) Nagod (b) Barabanki
(a) Vietnam (b) France (c) Faizabad (d) Gorakhpur
(c) Seychelles (d) Singapore 195. The first Russia India Strategic Dialogue will take
place in which city?
186. President Ram Nath Kovind inaugurated Gyan (a) Mumbai (b) NewDelhi
Kumbh in which city? (c) Moscow (d) St Petersburg
(a) Gorakhpur (b) Lucknow
196. RBI has initiated steps to set up a wide-based
(c) Rishikesh (d) Haridwar
digital PCR to capture details of all borrowers,
including wilful defaulters and also the pending
187. The first Global Conference on Air Pollution and
legal suits in order to check financial
Health will be held at which city?
delinquencies.What is “R” is PCR?
(a) Mumbai (b) Geneva
(a) Registry (b) Recognition
(c) Paris (d) California
(c) Revenue (d) Rebate
188. When is International Day to End Impunity for
197. Who was elected as president of the International
Crimes against Journalist celebrated?
Boxing Association (AIBA)?
(a) 4th Nov (b) 3rd Nov
(a) Thomas Bach (b) Gafur Rakhimov
(c) 2nd Nov (d) 1st Nov
(c) Nicolas Djuvone (d) Maria Draghi
189. The National Green Tribunal has ordered a 4
198. Who was chosen for the prestigious Raja Ram
member committee to conduct an inspection near
Mohan Roy award?
the Polavaram Dam.Where is Polavaram Dam (a) Kedarnath Sharma (b) Mihir Singh
situated? (c) Subash Paul (d) N Ram
(a) Tamil Nadu (b) Karnataka
(c) Andhra Pradesh (d) Uttar Pradesh 199. When is World Tsunami Awareness Day
observed?
190. Who is the author of “Notes of a Dream: The (a) Nov 3 (b) Nov 1
Authorized Biography of AR Rahman”? (c) Nov 5 (d) Nov 6
(a) Trilok Sharma (b) Krishna Trilok
(c) Arundhati Roy (d) Manvendra Singh 200. Which organization launched UTS application?
(a) Indian Navy (b) Indian Army
191. 18th Indian Ocean Rim Association (IORA) (c) Indian Railways (d) Indian Air Force
Council of Ministers Meeting held in which city?
(a) Mumbai (b) Durban
(c) Cape Town (d) New York

©LegalEdge Tutorials Page 46 of 60


Replication or other unauthorized use of this material is prohibited by the copyright laws of India
ANSWER KEY AND EXPLANATIONS
Prep Test – Logical Reasoning

1. c 2. b 3. d 4. c 5. d 6. d 7. d 8. a 9. d 10. b
11. c 12. d 13. c 14. d 15. d 16. c 17. c 18. b 19. c 20. d
21. d 22. a 23. d 24. c 25. c 26. b 27. d 28. c 29. d 30. d
31. a 32. c 33. a 34. d 35. c 36. d 37. d 38. a 39. d 40. b

1. (c) Statement II proves that the suggested measure is


Nehru Gandhi No not feasible.
Tall Short Glasses Statement III shows that a minority segment of the
House House glasses
A √ √ √ city traffic is made up of individual car owners
B √ √ √ and that their reduction will not make a
difference.
C √ √ √
6. (d) We cannot say that ' a person below poverty
D √ √ √ line is capable of doing any job or not, also
E √ √ √ problem of malnutrition can be handled easily by
F √ √ √ various welfare schemes. So argument I is not
B and E are short and not wearing glasses. strong. Argument II is completely illogical, hence
2. (b) also rejected. Therefore answer is option d.
Delhi Bangalore Tall Short Girls Boys 7. (d) Both the arguments are way out of track and
A √ √ √ do not connect to the major objective, hence both
B √ √ √ the arguments are weak. therefor answer is option
d.
C √ √ √
8. (a) ‘173’ or ‘371’ appear three times on total.
D √ √ √
9. (d) ‘even-even-odd’ appears four times in the
E √ √ √ series.
F √ √ √ 10. (b)
Clearly, D is the tall girl from Bangalore. ‘163’ appears two times.
3. (d) ‘169’ does not appear.
Green Yellow New Old Law Medical ‘563’ does not appear.
Cover Cover Volume Volume Reports Extracts ‘569’ appears one times.
A √ √ √ In total required sequence appears three times.
11. (c) Required sequence appears two times i.e. 3 – 25
B √ √ √
- 18 and 21 – 45 - 46.
C √ √ √ 12. (d) If all the alphabets are dropped from the given
D √ √ √ arrangement, 3 will be the sixth from left end.
E √ √ √ 13. (c)
F √ √ √
Clearly, E and F are old volumes which have green
covers and are medical extracts.
4. (c) Two dancers are Shailja and Tanuja.
Two violinists are Girija and Vanaja. Since
Jalaja and Shailja cannot be violinists, so remaining
violinist is Tanuja. 14. (d)
The four vocal musicians are Vanaja, Jalaja, Eighth to the left of fifteenth element from the
Shailja and Tanuja. right= 8+15= 23rd element from the right which is ‘Z’
Clearly, Tanuja is a violinist, a vocal musician and 15. (d)
a dancer.
5. (d) In this case, all of the above statements weaken
16. (c)
the administration's claim.
I N T E L L I G E N T
Statement I proves that there are other measures
F K Q B I I F D B K Q
that might be more effective.
(-3) (-3) (-3) (-3) (-3) (-3) (-3) (-3) (-3) (-3) (-3)

©LegalEdge Tutorials Page 47 of 60


Replication or other unauthorized use of this material is prohibited by the copyright laws of India
Similarly ENTERTAINMENT will be coded as
BKQBOQXFKJBKQ. So option C.
17. (c)

Similarly DISTRIBUTION will be coded as


P is uncle of R.
STTIBUDIONRI
23. (d)
18. (b):
Let (+) denotes male person & (-) denotes female
person.
None of the options implies.
24. (c)

Option (3) is one of the implication.


25. (c)
S is maternal uncle of Y Given: X - Y + Z ⇒ X = Y ≠ Z According to
19. (c) option (1) X ≠ Y ≠ Z (2) X < Y = Z
Let (+) denotes male person & (-) denotes (3) X = Y < Z option (3) is one of the
female person. implication.
26. (b)

option (2) is one of the implication.


27. (d):
Relation of M and N can’t be determined. X=Y–Z ⇒X>Y=Z
Hence, Data Inadequate. According to option (1) Y < X > Z (2) X < Y < Z
20. (d): (3) Z < X > Y
Let (+) denotes male person & (-) denotes None of the option is implication of given
female person. information
28. (c)

B is brother-in-law of A and A is male. Thus A


is also brother-in-law of B.
21. (d): From the above diagram its clear that:
Let (+) denotes male person & (-) denotes Only conclusion II follows
female person.
29. (d)

From the above diagram its clear that:


All conclusions follow.
We do not know the gender of B. Therefore,
30. (d)
can’t be determined.
22. (a):
Let (+) denotes male person & (-) denotes
female person.

©LegalEdge Tutorials Page 48 of 60


Replication or other unauthorized use of this material is prohibited by the copyright laws of India
From the given information, we can make the
following table:
Person Bike Position
Amar Pulsar 2
Bitu Apache 5
From the above diagram it’s clear that: None Chetan Splendor 4
follows Darshan Hayabusa 1
31. (a) Ehsaan Unicorn 3
Hence Chetan was driving Splendor.
36. (d) From the given information, we can make the
following arrangements:
D E A F B C
Or
From the above diagram its clear that: B C F A E D
None follows Hence B or C can sit at the other extreme end. So
32. (c) answer is option d.
From the first statement, Orange and Pineapple 37. (d):
are in one box and watermelon is in the other box. In the first option B1 and B6 are together. Hence it
From statement B, Mango and Strawberry is in the is false.
box with watermelon. From third statement, In option 2, as B1 and B5 have to be together, so it
blueberry and guava are with orange and is again false.
In option 3, G1 G3 should be together, hence it is
pineapple and Apple is with watermelon. Hence
again false.
the fruits in box 1 are Orange, pineapple, Option 4 satisfies all the conditions and hence is
blueberry and guava and in box 2 are watermelon, the answer.
mango, strawberry and apple. 38. (a) The word ’is’ is present in each of the sentences
33. (a): and the code present in all the codings is ‘see’
From the given information we can have the Hence the code of ‘is’ is ‘see’. In the first two
following two cases: sentences ‘it’ is common word and the common
Case I: the order is N > L > O > Q > __ > P code is ‘nee’. So the code of ‘it’ is ‘nee’. So the code
Case II: the order is N > Q > O > L > P. of ‘who’ is ‘tim and that of ‘where’ is ‘ble’.
In both the cases the positions of other persons is According to options only Option (A) has the
not clear. But it can be inferred from the words “tim ble see".
information that none of Qutab, Omprakash, Hence option (A) is the correct answer
Lakshman, Raman, Mahendra and Pretam is 39. (d)
tallest. Hence the tallest person is Narendar. From the given information, we can arrange the
34. (d): dwarfs from left to right as:
From the given information, we can make the dreamy sleepy bashful sneezy dopey grumpy
following table: happy. Hence dopey is standing just before
Basket Athlet Swim Badmi grumpy.
Tennis Hockey
ball ics ming nton 40. (b) From the given information we can make the
Saurav √` √`` √ following family tree.
Kartik √ √ √ √
Manpreet √ √ √ √
Lokesh √ √ √ √ √
Nikhil √ √ √
Ganesh √ √ √
Prakash √ √ √
Rohit √ √
From the table it is clear that Lokesh plays tennis,
swimming and badminton. Hence he will be
As we do not know Sheeba’s gender, thus options
selected.
1, 3 and 4 are false.
35. (c):
Hence the answer is option b.

©LegalEdge Tutorials Page 49 of 60


Replication or other unauthorized use of this material is prohibited by the copyright laws of India
ANSWER KEY AND EXPLANATIONS
Prep Test – English Language

1. B and E 2. D and F 3. C and E 4. B and F 5. E and F 6. C and F 7. B and E 8. D and F 9. C and E 10. AandB
11. d 12. b 13. d 14. a 15. a 16. d 17. b 18. d 19. c 20. b
21. f 22. b 23. b 24. d 25. c 26. b 27. a 28. b 29. a 30. b
31. b 32. a 33. b 34. c 35. c 36. b 37. c 38. c 39. d 40. d

1. B and E The trigger despite indicates that the reefs had placate and appease can mean to calm, so choices (E) and
not justified the environmentalist’s initial fears, so you (F) give you appropriate, equivalent sentences.
need a word such as positive or harmless in the blank. 6. C and F If they believe the benefits of the energy-
Neither caustic nor interminable means positive or producing ingredients increase proportionally, the
harmless, so eliminate answer choices (C) and (E). athletes would want to eat more of the energy bars, so
Neither unfounded, which means groundless, nor you need a word that means something like plentiful.
plaintive, which means mournful, makes sense in the Both scant and measured are nearly the opposite of what
blank; eliminate choices (A) and (F).Both benign and you’re looking for, so eliminate choices (A) and (E).
innocuous can mean harmless, so choices (B) and (E) give Neither furtive, which means stealthy, nor solvent, which
you appropriate, equivalent sentences. means capable of paying debts, makes sense in the blank,
2. D and F Recycle the clue and put enthusiasm in the blank. so eliminate choices (B) and (D). Both copious and
Both indifference and heedlessness are nearly the profuse can mean plentiful, so choices (C) and (F) give
opposite of what you are looking for, so eliminate choices you appropriate, equivalent sentences.
(B) and (E). Neither cessation, which means stoppage, 7. B and E The bill is described as comprehensive and has
nor rhetoric, which is the art of elective or persuasive use 249 new regulations on a single industry, so something
of language, make sense in the blank, so eliminate choices like full of would make sense in the blank. Both deficient
(A) and (C). Both fervency and zeal can mean in and unencumbered by are nearly the opposite of what
enthusiasm, so answer choices (D) and (F) give you you’re looking for, so eliminate choices (C) and (D).
appropriate, equivalent sentences. Neither elucidated by, which means clarified by, nor
3. C and E The clue innumerable fruitless attempts exempted from makes sense in the blank, so eliminate
indicates that Allen lacked the crucial qualification to choices (A) and (F). Both rife with and replete with mean
juggle, so a word that means something like manually full of, so choices (B) and (E) give you appropriate,
coordinated will it the blank. Both lumbering and equivalent sentences.
awkward are nearly the opposite of what you’re looking 8. D and F Recycle the clue embarrassing into the blank.
for, so eliminate choices (B) and (F). Neither sedate, Both assuaging and bolstering are nearly the opposite of
which means calm, nor implicit, which means implied, what you’re looking for, so eliminate choices (B) and (E).
make sense in the blank, so eliminate choices (A) and (D). The impact may be monumental, but that answer choice
Both dexterous and implicit can mean manually isn’t succinctly negative, so eliminate choice (C); virulent,
coordinated, so choices (C) and (E) give you appropriate, on the other hand is too strong, so eliminate choice (A).
equivalent sentences. Both discomfiting and mortifying can mean
4. B and F If Alexander’s generals broke up his empire at embarrassing, so choices (D) and (F) give you
his death, then its cohesion must have been weak or appropriate, equivalent sentences.
short-lived. Both abiding and protracted are nearly the 9. C and E Samuel Huntington’s ideas are still passionately
opposite of what you’re looking for, so eliminate choices debated so they must be described as ideas that people
(A) and (C). Neither redoubled, which means made twice are likely to disagree about. His opinions may well have
as great, nor renowned, which means famous, make been pedantic or hegemonic, but those words do not
sense in the blank, so eliminate choices (D) and (E). Both necessarily lead to disagreement. The same goes for
precarious and tenuous can mean weak, so choices (B) zealous, which, in addition, is also used to describe
and (F) give you appropriate, equivalent sentences. people, rather than their ideas. The correct answers are
5. E and F If you’re comfortable with the meaning of choices (C) and (E), since polemical means arguing
propitiatory, you can recycle the verb propitiate into the passionately, and divisive means causing a
blank; if not, the clue icy stare and aloof demeanor can disagreement.
tell you—like Johann—that the gifts didn’t calm her 10. A and B The sentence tells you that there was no plant
anger. Both exacerbate and aggravate are nearly the life in the area before, based on the time trigger word
opposite of what you’re looking for, so eliminate choices previously. Furthermore, if it is being implied that since
(C) and (D). Neither vilify, which means to speak ill of, plants have been able to generate and grow, humans will
nor garner, which means to amass or acquire, make sense be able to live in the area some day, then you know that
in the blank, so eliminate choices (A) and (B). Both plant life being described, must be alive and flourishing.

©LegalEdge Tutorials Page 50 of 60


Replication or other unauthorized use of this material is prohibited by the copyright laws of India
This eliminates static, waning, and flagging. Fervent is affected. There is no support for choice c. Choice d can be
used to describe human feelings or something that is very ruled out because there is nothing to indicate that DDT
hot, so it does not work in this context. Incipient and and toxaphene are the most toxic. Choice e is illogical.
nascent both mean that the plant life has recently come 15. The second and third sentence combine to give support
into being and are the correct answers. to choice a. The statement stresses that there must be a
11. The passage speaks specifcally about onions and bay judge’s approval (i.e., legal authorization) before a search
leaves in clam chowder—not about soups in general, and can be conducted. Choices b and d are wrong because it
thus choice b is not correct. No mention is made to is not enough for the police to have direct evidence or a
nutrition or poison, nor does the passage address over- reasonable belief—a judge must authorize the search for
it to be legal. Choices c and e are not mentioned in the
cooked clams.
passage.
12. The last sentence in the paragraph clearly supports the
16-31 Inference from the concepts of English Grammar.
idea that the renewed interest in Shakespeare is due to
32. The central idea of the passage is not about rebirth and
the development of his characters. Choice a is incorrect God, changes and humanity have only part a part of the
because the writer never makes this type of comparison. concept but the central idea remains philosophy.
Choice c is wrong, because even though scholars are 33. Renaissance means rebirth but it is just rebirth in the
mentioned in the paragraph, there is no indication that passage not of God or philosophy. Changes in humanity
the scholars are compiling the anthology. Choice d is were seen but that is not Renaissance, so the answer is
wrong because there is no support to show that most (b).
New 34. Statements (a), (b) and (d) are clearly mentioned in the
Yorkers are interested in this work. There is no support paragraph, but regarding education, it says mostly men
for choice e. not only men.
13. This answer is implied by the whole paragraph. The 35. As is evident from the first 2 lines of the passage, the
author stresses the need to read critically by performing answer is (c).
operations on the text in a slow and specific manner. 36. (b) (a), (c) and (d) have been mentioned in the passage
Choice a is incorrect because the author never says that but Samina was doing all this because of her age is
reading is dull. Choices b, c, and e are not supported by nowhere mentioned.
the paragraph. 37. He changed her attitude and made her mother happy, so
14. The support for this choice is in the second sentence, after going through her results she felt guilty.
which states that in some countries, toxic insecticides are 38. It is teaching us a lesson at the end, it is a moral story.
still legal. Choice b is incorrect because even though 39. The lesson taught in the story.
polar regions are mentioned in the paragraph, there is no 40. The sound that Cows make is called 'bellow'.
support for the idea that warmer regions are not just as

©LegalEdge Tutorials Page 51 of 60


Replication or other unauthorized use of this material is prohibited by the copyright laws of India
Answer Key & Explanations (CAPS)

1. b 2. a 3. b 4. d 5. a 6. d 7. c 8. c 9. d 10. a
11. c 12. b 13. a 14. c 15. d 16. c 17. b 18. a 19. a 20. d
21. b 22. d 23. b 24. c 25. d 26. c 27. a 28. d 29. c 30. d
31. b 32. d 33. b 34. d 35. a 36. b 37. d 38. c 39. b 40. d
41. c 42. d 43. b 44. d 45. a 46. d 47. a 48. a 49. d 50. a
51. b 52. b 53. c 54. a 55. a 56. c 57. d 58. c 59. a 60. d
61. a 62. c 63. d 64. d 65. d 66. a 67. c 68. d 69. a 70. d
71. b 72. c 73. d 74. c 75. a 76. b 77. d 78. b 79. c 80. d
81. a 82. d 83. d 84. a 85. b 86. b 87. b 88. b 89. a 90. c
91. b 92. a 93. a 94. b 95. c 96. a 97. d 98. a 99. d 100. b
101. a 102. c 103. d 104. b 105. a 106. a 107. d 108. c 109. d 110. b
111. d 112. c 113. c 114. d 115. b 116. d 117. b 118. c 119. c 120. d
121. a 122. c 123. b 124. b 125. c 126. d 127. a 128. b 129. b 130. a
131. c 132. a 133. b 134. a 135. a 136. a 137. a 138. c 139. d 140. b
141. a 142. b 143. a 144. b 145. a 146. c 147. c 148. a 149. d 150. b
151. a 152. d 153. c 154. b 155. a 156. d 157. a 158. c 159. a 160. c
161. c 162. a 163. d 164. a 165. c 166. d 167. a 168. c 169. a 170. c
171. a 172. d 173. a 174. b 175. a 176. c 177. c 178. c 179. a 180. b
181. d 182. b 183. b 184. b 185. a 186. d 187. b 188. c 189. c 190. b
191. b 192. d 193. d 194. c 195. d 196. a 197. b 198. d 199. c 200. c

1. Andhra Pradesh has topped the list of states with the disability sector, education, trade-investment, scientific
highest employability followed by Rajasthan and cooperation, agriculture and on higher education
Haryana, as per the India Skills Report 2019. focussing on PhD by both the nations.
2. Paytm (One 97 Communications Limited) has partnered 10. HRD Minister Prakash Javadekar launched the
with the Life Insurance Corporation of India (LIC) to Institution’s Innovation Council through video
offer online insurance premium payments on its conferencing in New Delhi. The purpose of formation of
platform. network of Institution’s Innovation Councils (IICs) is to
3. The Governor of Tripura Prof. Kaptan Singh Solanki encourage, inspire and nurture young students by
inaugurated the ‘7th International Tourism Mart for the exposing them to new ideas.
North-East States in the presence of Minister of State 11. Laos is a Southeast Asian country traversed by the
(I/C) for Tourism K.J Alphons and the Chief Minister of Mekong River and known for mountainous terrain,
Tripura Biplab Kumar Deb in Agartala, Tripura. French colonial architecture, hill tribe settlements and
4. United Nation Environment has awarded Wildlife Crime Buddhist monasteries.
Control Bureau (WCCB), Ministry of Environment, 12. The fourth ‘UK India Business Council Ease of Doing
Forest and Climate Change, Government of India with Business Report’ has been released. It highlighted that
Asia Environment Enforcement Awards, 2018 for UK businesses’ perceptions of corruption as a barrier to
excellent work done by the Bureau in combating operating in India has halved since 2015. An in-depth
transboundary environmental crime. survey of businesses with a combined turnover of USD
5. Azim Premji, chairman of Wipro, has won Forbes India 122 billion, the report found only 25% of UK businesses
Leadership Awards 2018, Lifetime Achievement. Wipro saw corruption as a major barrier in 2018, as opposed to
is India’s fourth-biggest software services company by 51% in 2015.
revenue and Ajim Premji is the second richest Indian. 13. The World Health Organisation (WHO) in a report stated
6. Prime Minister Shri Narendra Modi laid the Foundation that India is the only country to have recorded a
Stones of City Gas Distribution (CGD) Projects in 65 substantial decline in malaria cases in 2017 out of the 11
Geographical Areas (GAs) in 129 Districts under the 9th highest burden countries worldwide. India registered a
CGD Bidding Round recently awarded by Petroleum 24% reduction in cases over 2016, largely due to
and Natural Gas Regulatory Board (PNGRB) substantial declines of the disease in the highly malarious
7. The second edition of the India-UAE Strategic Conclave state of Odisha, home to approximately 40 percent of all
will be held in the capital city of Abu Dhabi on 27th of malaria cases in the country. According to the report,
November. The day-long conclave will initiate a dialogue India accounted for 4% of global malaria cases in 2017.
to widen the scope of bilateral investments. 14. Centre for Science and Environment (CSE) has won this
8. Dharmendra Pradhan is the present Petroleum Minister year’s Indira Gandhi Prize for Peace, Disarmament and
of India. Development, an award conferred by Indira Gandhi
9. After visiting Vietnam, President of India Ram Nath Trust each year on the late Prime Minister’s birth
Kovind was in Australia on his second leg of 2-Nation anniversary.
visit. He met Australian Prime Minister Scott Morrison in 15. State-run Airports Authority of India (AAI) has inked an
Sydney. He became the first Indian President to visit agreement with the US Trade and Development Agency
Australia. 5 MoUs were exchanged in the field of
©LegalEdge Tutorials Page 52 of 60
Replication or other unauthorized use of this material is prohibited by the copyright laws of India
(USTDA) to develop a roadmap for modernization of its ‘Radio Kashmir-In Times of Peace & War’ written by Dr
air traffic services. Rajesh Bhat.
16. Voreqe Bainimarama has Sworn in as Fiji’s Prime 28. National Stock Exchange of India (NSE) has launched a
Minister for four more years after winning an election mobile application and web-based platform for retail
with a reduced majority. Voreqe Bainimarama was investors to buy government securities. The new app –
affirmed prime minister in a ceremony in the capital Suva NSE goBID, was launched by Sebi chairman Ajay Tyagi.
after his FijiFirst Party won just over half the votes in the 29. Switzerland has topped the list of the IMD World Talent
recent election. Ranking 2018 for the fifth year in a row. Among the
17. Centre for Science and Environment is a not-for-profit BRICS countries, South Africa remains in the middle
public interest research and advocacy organisation based position (50th) performing better than India (53rd) and
in New Delhi. Brazil (58th) but lagging behind China (39th) and Russia
18. The United Nations children’s agency UNICEF (46th).
appointed Millie Bobby Brown, the star of hit Netflix 30. United Nations Universal Children’s Day was
series “Stranger Things”, as its youngest ever goodwill established in 1954 and is celebrated on November 20th
ambassador on Tuesday and the 14-year-old had a each year to promote international togetherness,
message for world leaders: “Listen to us.” The awareness among children worldwide, and improving
appointment – marked on World Children’s Day at children’s welfare.
United Nations Headquarters and the Empire State 31. Bangladesh Krida Shiksha Prothishtan (BKSP) won the
Building in New York – makes the 14-year-old UNICEF’s 59th edition of the junior boys Subroto Cup International
youngest-ever Goodwill Ambassador. Football Tournament beating Amini School, Afghanistan
19. The International Cricket Council (ICC) announced its by a solitary goal at the Ambedkar Stadium in New
partnership with Google to offer the fans a great viewing Delhi.
experience of the upcoming semi-finals and final of the 32. The Maldives cabinet has approved to rejoin the
ICC Women’s World T20 to be held in West Indies. The Commonwealth, two years after withdrawing from the
collaboration includes enabling the casting of video 53-nation grouping. The announcement was made by
content from the ICC mobile app on cast-enabled New President of Maldives Ibrahim Mohamed Solih.
devices, like Chromecast and Google Home Hub. Now, the proposal will be sent to Parliament for
20. South Korea’s Kim Jong-yang has been elected as ratification.
Interpol’s next president, edging out a longtime veteran 33. India’s first specialized hospital for elephants has been
of Russia’s security services, Alexander Prokopchuk, opened by Agra Divisional Commissioner Anil Kumar at
who was strongly opposed by the US, Britain and other Farah block’s Churmura village, Mathura.
European nations. 34. The Asia-Pacific Economic Cooperation (APEC) summit
21. The International Criminal Police Organization, more was held in Port Moresby, Papua New Guinean capital.
commonly known as Interpol, is the international This is the first time Papua New Guinea (PNG), the
organization that facilitates international police poorest of the 21 countries in APEC, has hosted the
cooperation. Headquarters: Lyon, France. summit.
22. The 12-day long joint military exercise between India and 35. The 14th edition of JashneBachpan, a theatre festival for
the United States known as ‘Vajra Prahar’ commenced in children, will see performances from Switzerland, Sri
Jaipur. Lanka and Indonesia along with their Indian
23. The 12-day long joint military exercise between India and counterparts in New Delhi.
the United States known as ‘Vajra Prahar’ commenced in 36. India’s top junior shuttler Lakshya Sen (17 years old)
Jaipur. A contingent of the United States Pacific won bronze medal at Li Ning BWF World Junior
Commander arrived at Mahajan Field Firing Ranges for Badminton Championships 2018 held at Markham,
undertaking Joint training with the Indian Special Forces. Canada.
24. Andhra Pradesh is building a world-class and India’s 37. The purpose of Africa Industrialization Day (AID),
first ‘Justice City’ within its new capital Amaravati with marked each year on 20 November, is to raise global
the vision to provide a state-of-the-art ecosystem built on awareness of the challenges faced by the continent with
latest technology for supporting the judicial system in the regard to industrialization.
state. 38. The 49th edition of the International Film Festival of
25. The 27th BASIC Ministerial Meeting on Climate Change India opened with the gala opening ceremony at the
was held in New Delhi, India. The meeting was chaired Shyama Prasad Mukherjee Indoor Stadium in Goa.
by H.E. Dr Harsh Vardhan, Minister of Environment, 39. Israel is the focus country, Jharkhand is the focus state
Forest and Climate Change of India. this year. Ten Israeli films including The Other Story will
26. Indian-American Shantanu Narayen, the Chief Executive be screened at the IFFI.
Officer (CEO) of Adobe, has been named by Fortune in 40. The World Toilet Day is organized around the world on
its 2018 Business Person of the Year list. The list ranks 20 19th of November. The day is about inspiring action to
business executives “delivering on the bottom line and tackle the global sanitation crisis. The theme for WTD
beyond”. Narayen ranks 12th on the list. 2018 is ‘When Nature Calls’.
27. Minister of State for Development of North Eastern 41. Leprosy, also known as Hansen’s disease, is a bacterial
Region (I/C) Dr Jitendra Singh released a book titled disease which affects the skin and nerves which can lead
to physical deformity and disability if left untreated.

©LegalEdge Tutorials Page 53 of 60


Replication or other unauthorized use of this material is prohibited by the copyright laws of India
Despite a centuries-long stigma, it is not hereditary, it is pursue structural integration. It is an intergovernmental,
completely curable, and is only mildly infectious – more binding, permanent framework and a structured process
than 85% of cases are non-infectious and over 95% of the to gradually deepen defence cooperation within EU
population has a natural immunity to the disease. framework. PESCO is also open to NATO members also
42. The mountain gorilla (Gorilla beringei beringei) is one of who wish to take part in it. There is no provision about
the two subspecies of the eastern gorilla. The subspecies the inclusion of non-EU NATO member. Denmark,
is listed as endangered by the IUCN, with only two which has anopt out from EU defence matters, and Malta,
surviving populations. One is found in the Virunga were the only EU countries not to sign up, along with
Mountains of East Africa in three bordering national Brexiting Britain. The participating Member States are:
parks: Mgahinga Gorilla National Park in Uganda, Austria, Belgium, Bulgaria, Czech Republic, Croatia,
Volcanoes National Park in Rwanda, and Virunga Cyprus, Estonia, Finland, France, Germany, Greece,
National Park in the Democratic Republic of Congo Hungary, Italy, Ireland, Latvia, Lithuania, Luxembourg,
(DRC). The other population is found in Uganda's the Netherlands, Poland, Portugal, Romania, Slovenia,
Bwindi Impenetrable National Park. A count in 2018 put Slovakia, Spain and Sweden.
the mountain gorilla population at just over 1,000. The 51. In order to protect and increase wild tiger population,
species was previously listed as critically endangered nearly one million acres of protected habitat in India and
until 2018. Bhutan will be covered under a new private conservation
43. Rothschild’s giraffe has gone from endangered to near effort. The 'Project C.A.T - Conserving acres for Tigers' by
threatened. The International Union for Conservation of Discovery Communications and NGO World Wildlife
Nature (IUCN) Red List classifies species according to Fund (WWF) aims to conserve the wild tiger population,
their extinction risks on a spectrum from “extinct” to which has dropped by 96 per cent in the last century
“least concern.” alone to only 4,000 left in the wild due to habitat loss and
44. It is located in Zambia pervasive poaching. This transboundary partnership will
45. It was fought between the Andamanese and the British allow rangers to more closely monitor tiger health and
The Battle of Aberdeen, on the Andaman Islands close to other key scientific data, take additional anti-poaching
Port Blair, was an armed conflict that occurred on May safeguards, and maintain land and corridors to improve
14, 1859 between the natives of the Andaman islands, movement of all wild animals. Discovery will also
armed with arrows and spears, and the gun-bearing leverage its formidable creative capabilities and global
officers and to some extent the convicts of the penal multiplatform distribution of channels to 3 billion
settlement. There had been skirmishes with the British cumulative worldwide viewers to support WWF's Tx2
right from 1857 when the penal settlement was effort, which began in 2010, to double the population of
established. wild tigers across the world by 2022.
46. The Marshall Islands is convening an entirely online 52. Solution (b) An area of 299.52 sq kms has been declared
Virtual Summit of the Climate Vulnerable Forum on 22 as the Aghanashini Lion-tailed Macaque Conservation
November. The Virtual Climate Summit is the brainchild Reserve in Karnataka.
of Marshall Islands President Hilda Heine. 53. Solution (c) The Prime Minister of India launched a
47. The Climate Vulnerable Forum (CVF) is a global Grand Challenge on resolving seven identified Ease of
partnership of countries that are disproportionately Doing Business problems with the use of cutting edge
affected by the consequences of global warming. The technologies. The objective of this challenge is to invite
forum addresses the negative effects of global warming innovative ideas based on Artificial Intelligence, Internet
as a result of heightened socioeconomic and of Things, Big Data Analytics, Blockchain and other
environmental vulnerabilities. These countries actively cutting edge technology to reform Government
seek a firm and urgent resolution to the current processes. The platform for the Grand Challenge is the
intensification of climate change, domestically and Startup India Portal.
internationally. 54. Solution (a) Under Basel III, the minimum capital
48. Statement 1 - The epistemology of Vaiśeṣika school of adequacy ratio that banks must maintain is 8%. The
Hinduism, like Buddhism, accepted only two reliable capital adequacy ratio measures a bank's capital in
means to knowledge: perception and inference. relation to its risk-weighted assets. The capital-to-risk-
Statement 2 – Charvaka deals with Materialism. weighted-assets ratio promotes financial stability and
49. Green Growth Equity Fund  India and the UK efficiency in economic systems throughout the world.
announced the launch of an Early Market Engagement 55. Solution (a)
for the joint UK-India Fund, namely a Green Growth 56. UN selects Noida, Greater Noida to participate in Global
Equity Fund  It aims to leverage private sector Sustainable Cities 2025 initiative.
investment from the City of London to invest in green 57. The two Indian movies competing for the honour are
infrastructure projects in India  Both governments will Tamil film Baaram and Ladakhi movie walking with the
invest up to £120 million each (i.e. totally £ 240 million)  Wind.
The joint fund which will be established under the NIIF 58. Australia won the ICC Women’s World T20 title for the
framework. fourth time when they defeated England by eight wickets
50. Permanent Structured Cooperation (PESCO). It is the in the final at North Sound, Antigua.
part of the European Union's (EU) Common Security and 59. India to Lay Foundation Stone for Kartarpur Corridor
Defence Policy (CSDP) in which 25 national armed forces Route.

©LegalEdge Tutorials Page 54 of 60


Replication or other unauthorized use of this material is prohibited by the copyright laws of India
60. India has achieved a new spot on the Guinness World Mahatma Gamdhi at the Jubilee Park in the town of
record as 2,353 people, led by Bollywood actress and Parramatta, which is a bustling suburb of Sydney.
fitness enthusiast Shilpa Shetty Kundra in Pune held 77. 12th World Congress on mountain Medicine started in
the abdominal plank position for 60 seconds Kathmandu, the capital of Nepal. The theme of 4 day
simultaneously. congress is “Mountain Medicine in the Heart of the
61. India’s economic growth will slow down somewhat Himalayas”.
but remain robust, at close to 7.50 percent in 2019 and 78. The United Nations children’s agency UNICEF
2020,the Organisation for economic Cooperation and appointed Millie Bobby Brown, the star of hit Netflix
Development(OECD) has said in its 2018 Economic series “Stranger Things”, as its youngest ever
Outlook. goodwill ambassador and the 14-year-old had a
62. Australia bagged their 4th Women’s World T20 crown message for world leaders: “Listen to us.”
after a thumping eight wicket victory over England at 79. November 21, 2018 is the 46th annual World Hello
the Sir Vivian Richards Stadium in North Sound, day.
Antigua and Barbuda. 80. The 27th BASIC Ministerial Meeting on Climate
63. India’s CA Bhavani Devi won the gold medal in the Change has urged the advanced countries to enhance
Sabre event in the Senior Commonwealth Fencing their support to the developing nations in effectively
Championship 2018 held in Canberra, Australia. implementing the Paris Agreement to combat climate
64. Pravin Tambe registered history in the T10 cricket change. The BASIC countries comprise Brazil, South
league, where he stunned Kerala Knights with a five- Africa, India and China. The two-day Meeting on
wicket haul, including a hat trick Climate Change concluded in New Delhi.
65. Magician Ricky Jay, who also acted in films and TV 81. India slipped two positions to rank at 53rd position in
shows such as “Boogie Nights”, “House of Games” a global survey released by IMD Business School
and “Deadwood”, is dead at 72. Switzerland on fostering and attracting talent. In 2017,
66. The United Nations General Assembly has designated India was ranked at 55th position.Switzerland topped
November 25 as the International Day for the the list.
Elimination of Violence against Women. The theme 82. Prime Minister Narendra Modi laid the foundation
was “Orange the World: #HearMeToo”. stone of Vishwakarma Skill University at Village
67. Maldivian Foreign Minister Abdulla Shahid has Dudhola in Palwal from Sultanpur village in
arrived in NewDelhi on a four-day visit to India. The Gurugram district.
Maldivian Foreign Minister is visiting India with the 83. Neighbourly, launched by Google, which is
aim to repair the bilateral ties which came under specifically designed for the Indian market, will give
severe strain during the previous government in the hyper local information on neighbourhoods.
island nation. 84. Bangladesh Krida Shiksha Prothisthan held their
68. Odisha Finance and Excise minister Sashi Bhusan nerves in a tense final and overcame Amini School,
Behera, in the presence of three of his cabinet Afghanistan, 1-0 to clinch the 59th edition of the
colleagues, inaugurated the Baliyatra Festival in Subroto Cup International Football Torurnament
Cuttack. (Junior Boy category).
69. Legendary boxer Mary Kom scripted history, 85. The International Criminal Police Organisation
becoming India’s only six time world boxing (INTERPOL) elected South Korean Kim Jong Yang as
champion after defeating Hanna Okhota of Ukraine. its new President at the 87th session of the INTERPOL
70. Odisha CM Naveen Patnaik released Men’s Hockey General Assembly in Dubai.
World Cup Anthem composed by renowned music 86. World Television Day is being celebrated on Nov 21
director and composer AR Rahman and penned by around the world
veteran lyricist Gulzar. 87. The 14th edition of international children’s theatre
71. The former Chelsea and Ivory Coast forward Didier festival, “Jashne Bachpan”, was inaugurated in New
Drogba has confirmed his retirement as a player. Delhi with folk dance performances from Assam,
72. The 50th Union World Conference on Lung Health Rajasthan, Punjab, and Manipur among other states.
will be held in Hyderabad. 88. RBI Central Board decided to constitute a Board for
73. Manipur Tourism,is once again returning with one of Financial Supervision (BFS) of the Central bank to
their most important yearly event, the Manipur Sangai examine the framework for banks under PCA.
festival. Celebrated each year, this is one of the most 89. The 12-day long joint military exercise between India
popular festivals of India. and the USA known as “Vajra Prahar”commenced in
74. Known for its hand-woven carpets around the world, Jaipur
Bhadohi received a major boost as the government has 90. The World Health Organisation (WHO) in a report
extended the “export excellence tag”. stated that India is the only country to have recorded
75. IT capital Bengaluru pays the highest salaries a substantial decline in malaria cases in 2017 out of the
followed by Mumbai and Delhi in India. The report is 11 highest burden countries worldwide.
based on a salary study done by LinkedIn. 91. Prime Minister Narendra Modi constituted a National
76. On the second day of this three-day visit to Australia, Implementation Committee(NIC), under the
President Ramnath Kovind unveiled a statue of Chairmanship of Union Home Minister Rajnath Singh
with Finance Minister and Minister of State for

©LegalEdge Tutorials Page 55 of 60


Replication or other unauthorized use of this material is prohibited by the copyright laws of India
Culture as Members, to commemorate 550th birth 106. National Press Day is celebrated on Nov 16 to
anniversary of Guru Nanak in the country and abroad. commemorate the establishment of the Press Council
92. The ministry of Civil Aviation has introduced the of India.
upgraded version of AirSewa 2.0 web portal and 107. In tune with its rise as an internet giant, China is
mobile app to provide superior user experience with roping in India as a niche digital partner, as part of
enhanced functionalities to air travellers. Beijing’s drive to cyber connect with Eurasia, its new
93. The world’s standard definitions of the kilogram,the frontier for trade and investments.
ampere,the Kelvin and the mole has been changed 108. The election commission has set “Sangwari” polling
after representatives from 60 countries voted to booths in various constituencies of Chattisgarh to
redefine the International System of Units(SI) for encourage women voters to exercise their franchise in
weight,current,temperature and amount of chemical the Assembly elections.
substance. 109. The Supreme Court has given nod for a separate High
94. Maharshtra Director General of Police Datta Court for Andhra Pradesh and it will start functioning
Padsalgikar unveiled “The Velvet Gloves”, a novel from January 1 next year.
written by former Intelligence Bureau (IB) officer 110. China will hold the 5th World Internet Conference in
Balakrishna Kamath, at a function in Mumbai. Wuzhen.This year’s WIC will be held in China’s
95. National Stock Exchange of India (N SE) launched a Zhejiang province under the theme of “Creating a
mobile application and web-based platform for retail digital world for mutual trust and collective
investors to buy government securities. The new app governance-towards a community with a shared
NSE goBID, was launched by Sebi chairman Ajay future in cyberspace”.
Tyagi. 111. China’s Xinhua news agency unveiled the world’s
96. The Maldives has approved to rejoin the first AI anchor that can read news in English and
Commenwealth, two years after withdrawing from Chinese.
the 53-nation grouping. 112. The Western Pacific nation of Palau has become the
97. Incumbent Fijian PM Voreque Bainimarama was first country to ban many kinds of sunscreen,in a
sworn in for a 2nd term after his ruling Fiji First Party move to protect its coral reefs from chemicals that
won this year’s general elections scientists say cause significant damage.
98. Centre for science and Environment (CSE) was 113. National Education Day of India is celebrated on 11
awarded this year’s Indira Gandhi Prize for Peace, November every year.
Disarmament and Development for its pioneering 114. Vice President Venkaiah Naidu said that India
work in the field of environmental education and contributed immensely to the war effort during the
protection and also for keeping the issue of World War I as he inaugurated the Indian War
environmental sustainability at the forefront of Memorial at Villers Guislain in France.
national attention and public policy. 115. The Kerala Government has initiated a scheme to
99. The United Nations’ (UN) Universal Children’s Day, provide free education and vocational training to the
which was established in 1954, is celebrated on children of those fishermen who died or went missing
November 20 each year to promote international in the 2017 Ockhi cyclonic storm.
togetherness and awareness among children 116. The Union Cabinet approved strategic sale of
worldwide.”Children are taking over and turning the government stake in Dredging Corporation of India to
world blue” is the theme of Universal Children’s Day, consortium of four ports.
2018 as announced by the United Nations. 117. Star Indian wrestler Bajrang Punia touched a new high
100. Leveraging its parcel business network,India Post in his career by achieving the number one rank in the
announced the soft launch of its e-commerce portal. world in the 65kg category..
101. The two-day summit to mark the World Tolerance 118. The 34th National Junior Athletic Championship was
Day was inaugurated in Dubai.Held for the first time held in Ranchi.
in UAE , the theme of the Summit was “Prospering 119. Restoration of an aristocratic house from a state of
from Pluralism:Embracing Diversity through partial ruin in Ladakh has won an UNESCO Asia
Innovation and Collaboration”. Pacific award for conservation.
102. Oxford Dictionary has declared the adjective “toxic” 120. Journalist Swati Chaturvedi was awarded the
as the word of the year for what it says reflects the reporters without Borders(RSF) Press Freedom
ethos, mood, or preoccupations in 2018. Award for Courage. Chaturvedi, who works as a
103. Credit rating agency Fitch affirmed India’s “Long freelance journalist, won the award for her book I am
Term Foreign Currency Issuer Default Rating(IDR)” at a Troll.
BBB- with a stable outlook. 121. Actress Manisha Koirala unveiled her book “Healed”,
104. Microsoft has announced to acquire a conversational her personal story of a battle against ovarian cancer.
Artificial Intelligence (AI) and bot development 122. The 25th edition of the Singapore India Maritime
company XOXCO for an undisclosed sum. Bilateral Exercise(SIMBEX),kicked off with gusto as
105. The World Tolerance Day 2018 was inaugurated on frontline ships of the Eastern Fleet of the Indian Navy
November 16 in Dubai, UAE.The theme was and those of the Republic of the Singapore Navy(RSN)
“Prospering from Pluralism: Embracing Diversity made their entry into the waters at port Blair.
through Innovation and Collaboration”.

©LegalEdge Tutorials Page 56 of 60


Replication or other unauthorized use of this material is prohibited by the copyright laws of India
123. The Cabinet approved setting up of a Central Tribal through an agreement treaty with the government of the
University in Andhra Pradesh. US.
124. RBI is likely to establish an ombudsman for digital 138. The Madras High Court on Wednesday directed the
payments by March to take the load off the increasing Union Ministry of Health and Family Welfare and the
number of complaints currently being handled by the competent authorities under the Drugs and Cosmetics
banking ombudsman. Act, 1940, to immediately stop online sale of medicines
125. India’s teenage shooting sensation Saurabh that could be sold only in pharmacies on the prescription
Chaudhary and Manu Bhaker clinched the gold medal of a registered medical practitioner.
at the 10m Air Pistol mixed team event on the last day 139. The Vice President of India, Shri M. Venkaiah Naidu
of the Asian Airgun Chanpionship in Kuwait. embarked on his first Africa visit from 31st October to
126. Harmanpreet Kaur became the first Indian to score a November 6, here today. In his weeklong visit, he will
century in T20 Internationals. visit Botswana, Zimbabwe and Malawi where he is
127. Indian opener Rohit Sharma became the first batsman scheduled to interact with heads of state of the three
to hit four centuries in T20 internationals during the nations, business fora and address Indian Community.
Lucknow clash against the West Indies, overtaking 140. The Morrison government has confirmed it will not sign
New Zealand cricketer Colin Munro, who has scored up to the United Nation’s migration pact, claiming it will
three centuries. undermine Australia’s harsh policies to deter asylum
128. India’s biggest social media summit,”Social Media seekers despite Australia’s role in helping to draft it.
Summit and Awards 2018”,kicked off in Vijaywada. 141. INS Tarangini, the first sail training ship of the Indian
129. Mathematician and Super30 founder Anand Kumar Navy, returned to Kochi after a 205-day voyage around
will be awarded for his “pioneering” contributions to the world under Lokayan-18. The ship visited 15 ports in
the field of education and illuminating the lives of 13 countries, traversed 22,000 nautical miles and trained
many talented students. Kumar will be honoured with a total of 120 trainees so that they would be ready to serve
“Global Education Award”on November 8 by the Indian Navy.
“Malabar Gold and Diamonds”. 142. The Social Welfare and Nutritious Meal Programme
130. National Cancer Awareness Day will be observed Department of the Government of Tamil Nadu under the
across the country on November 7 every year. able guidance of the Honble Chief Minister of Tamil
131. Explanation: The West Bengal government has Nadu had always ensured the welfare of the poor, the
decided to observe “Rosogolla Day” on November 14. down-trodden, Women, Children, Senior Citizens and
132. Google CEO Sundar Pichai sent an email to all trans-genders. Their health, nutrition, education,
Google employees Thursday saying the company has protection and development have been improved
fired 48 people over the last two years for sexual through various Social Welfare Schemes. They have also
harassment, including 13 "senior managers and above," been provided with innumerable opportunities and
and none received an exit package. facilities to live a dignified life.
133. Ethiopia's parliament has appointed the country's first 143. Indian Passport was recently declared the 66th most
female president, Sahle-Work Zewde, in a move hailed powerful password in the world according to the Global
as setting a new standard for women in the East African Passport Index ranking. The Passport Index is an online
nation, the Prime Minister's chief of staff announced tool which collects and displays the ranks of the
Thursday. passports of the world. It is the only real-time global
134. Pooja Dhanda, who grabbed a silver at the 2018 ranking of the world's passports updates frequently
Commonwealth Games, is the fourth Indian woman when new visa waivers and changes are implemented.
wrestler to have won a medal (all bronze) at the World 144. The World Bank, today released the Doing Business (DB)
Championships, following Alka Tomar (2006), Geeta Report, 2018. The Department of Industrial Policy and
Phogat (2012) and Babita Kumari Phogat (2012). Promotion (DIPP) is pleased to announce that India
135. India installed 4.9 GW of solar power, consolidating its ranks 100 among 190 countries assessed by the Doing
position as the second largest solar market in the world, Business Team. India has leapt 30 ranks over its rank of
during the first half of calendar year 2018. The country 130 in the Doing Business Report 2017.
was ranked second, following China during the January- 145. 01 November 2018 Current Affairs: The two frigates of
June period, according to a report by Mercom the Project 11356 class will be bought directly from
Communications India. Russia and 2 more will be built in an Indian Shipyard.
136. NATO's biggest military maneuvers since the Cold The two ships to be brought from Russia are expected to
War began in Norway in a hypothetical scenario be delivered by 2022 and have been designed to work
that involves restoring the Scandinavian country's with Ukraine-made gas turbines.
sovereignty after an attack by a "fictitious 146. The 49th International Film Festival of India (IFFI) 2018
aggressor. will be held in Goa from 20th to 28th November, 2018.
137. The headquarters of the United Nations is located at 750 49th edition of the festival will showcase 212 films from
United Nations Plaza, in Manhattan, New York City. The over 68 countries which reflects diversity of taste.
land occupied by this headquarters is not under the International Competition section has 15 films out of
administration of the US government, but the UN’s, since which 3 are Indian. Competition section itself represents
the area was converted into international territory films produced and co-produced by 22 countries.

©LegalEdge Tutorials Page 57 of 60


Replication or other unauthorized use of this material is prohibited by the copyright laws of India
147. Elon Musk had agreed to step down as chairman of retired England woman wicketkeeper-batter Claire
Electric carmaker Tesla as part of a settlement with the Taylor was also named in the ‘Hall of Fame’ during a
US Securities and Exchange Commission. Robyn ceremony in Dublin on Sunday.
Denholm will leave her Chief Financial Officer (CFO) 159. The National Cancer Awareness Day is observed on
post at Telstra and take over Musk’s position, which he’s which date in India? The National Cancer Awareness
been holding for 14 years. However, Musk will remain Day is observed every year in India on November 7 to
on the board as a director and Tesla CEO. stress upon the early detection and cure of cancer.
148. The British Broadcasting Corporation (BBC) has devised 160. A day before the Ekana International Cricket Stadium
a new campaign that is aimed at fighting back against in Lucknow was to host the first international T20
disinformation and fake news with a major focus on match the stadium has got a new name. On Monday
global media literacy, including workshops and debates the stadium was renamed after former prime minister
in countries like India. Atal Bihari Vajpayee.
149. There are currently over 4,800 known Trojan asteroids 161. Rohit Sharma sets record, becomes first cricketer in
associated with Jupiter. About 65% of these belong to the history to score 4 T20I hundreds. 07 November 2018
leading group (L4) located 60o in front of Jupiter in its Current Affairs: He achieved the milestone in 58 balls
orbit, while the other 35% cluster around the L5 during the second T20I against the Windies at the Bharat
Lagrangian point and trail 60o behind Jupiter. Ratna Shri Atal Bihari Vajpayee International Cricket
150. India lost a major trade dispute at the World Trade Stadium in Lucknow.
Organization (WTO) on Tuesday, after a dispute 162. The Supreme Court has given nod for a separate High
settlement panel largely upheld Japan’s complaint that Court for Andhra Pradesh and it will start functioning
New Delhi’s imposition of safeguard duty on imports of from January 1 next year. The new court, which will be
hot-rolled steel flat products during September 2015 and the 25th High Court in the country, will temporarily
March 2018 violated core global trade rules. function from a makeshift building till the permanent
151. China has unveiled its new-generation stealth combat structure is built in the Justice City complex in Andhra’s
unmanned aerial vehicle (UAV), stated to be the most de-facto capital Amaravati.
advanced drone developed so far by the country, 163. China unveiled on Tuesday a replica of its first
according to a media report Tuesday. permanently crewed space station, which would replace
152. India’s first domestically developed and built nuclear- the international community's orbiting laboratory and
powered ballistic missile submarine (SSBN), the symbolises the country's major ambitions beyond Earth.
INS Arihant, has completed its first deterrent patrol, 164. Veteran journalist and the Hindu Group of Publications
Indian Prime Minister Narendra Modi announced on Chairman N Ram has been chosen for the prestigious
November 5. Raja Ram Mohan Roy Award, presented by the Press
153. A plan to create the world’s largest marine sanctuary Council of India, for his outstanding contribution
in Antarctic waters was shot down when a key towards journalism, the council announced on Monday.
conservation summit failed to reach a consensus, with 165. Vice President M Venkaiah Naidu Saturday inaugurated
environmentalists on Saturday decrying a lack of the first India-bulit war memorial in northern France to
scientific foresight. pay tribute to thousands of Indian soldiers who fought
154. A sculpture in honour of Indian soldiers who fought selflessly and made supreme sacrifices during the World
during the First World War was unveiled on Sunday in War I. Naidu, who is on a three-day visit to Paris starting
the town of Smethwick in the West Midlands region of Friday, also interacted with French armed forces veterans
England on Sunday. and children at the inauguration of the Indian War
155. In December 2015, the UN General Memorial at Villers Guislain.
Assembly designated 5 November as World Tsunami 166. The Armistice of 11 November 1918 was
Awareness Day. the armistice that ended fighting on land, sea and air in
156. Delhi’s iconic Signature Bridge was inaugurated this World War I between the Allies and their opponent,
evening by chief minister Arvind Kejriwal. The 675- Germany.
metre Signature Bridge, which will be opened to public 167. Wrestler Bajrang Punia on Saturday touched a new
tomorrow, is an eight-lane carriageway that will reduce career high by achieving the number one rank in the
the travel time between north and northeast Delhi and world in the 65kg category.
share the burden of the Wazirabad Bridge. It will also 168. NDA’s Rafale deal was worth 7.878 billion Euros (Rs
connect the Outer Ring Road on the western bank of the 59,000 crore). The cost of 36 fighters was 3.402 billion
Yamuna river with Wazirabad Road on the eastern side. Euros (Rs 25,570 crore), each jet costing Rs 670-710 crore.
157. The Vietnamese capital of Hanoi will host a Formula One 169. Modi in Varanasi: PM inaugurates India's first multi-
Grand Prix for the first time in 2020. modal terminal on riverGanga. Prime
"This is a fulfilment of one of our primary goals," Minister Narendra Modi on Monday inaugurated
Formula One (F1) Commercial Managing Director Sean India's first multi-modal terminal on the Ganga river.
Bratches exclusively told CNBC at the announcement. He also received the country's first container cargo
158. Former India captain Rahul Dravid and Australian transported on inland waterways from Kolkata.
legend Ricky Ponting have been inducted into the ICC 170. Bangladesh ODI captain Mashrafe Bin Mortaza
Cricket Hall of Fame for their stupendous achievements yesterday received confirmation from ruling Awami
in international cricket. Along with Dravid and Ponting,

©LegalEdge Tutorials Page 58 of 60


Replication or other unauthorized use of this material is prohibited by the copyright laws of India
League to contest the seat from Narail-2 in his home Logistics(AVIAMAT-2018) at Air Force Auditorium,
district. Subroto Park, New Delhi
171. Children’s Day 2018: The nation is celebrating the 130th 183. India and Japan signed a Rs 1817 crore loan agreement
birth anniversary of the first prime minister of for the construction of the Turga Pumped storage hydel
independent India, Pt Nehru, who was born on power project at Purulia in West Bengal.
November 14, 1889, was known for his love for children 184. Qatar will host the 23rd edition of the World Corporate
and was fondly called Chacha Nehru. He once said: “The Games next year, which will provide an opportunity for
children of today will make the India of tomorrow. The corporations around the world to showcase their
way we bring them up will determine the future of the competitive prowess through sports.
country”. 185. Vietnam will host a Formula One Race in April 2020.
172. South African President Cyril Ramaphosa has accepted 186. President Ram Nath Kovind has stressed the need to
the invitation to be the chief guest at next year’s Republic impart quality education to the students. After
Day parade. inaugurating a two-day Gyan Kumbh at Patanjali
173. The United Nations Children's Fund-India (Unicef-India) Yogpeeth in Haridwar.
has appointed Asian Games gold-medalist sprinter Hima 187. The first Global Conference on Air Pollution and Health
Das as the country's first ever youth ambassador. will be held at WHO Headquarters in Geneva on 30
174. The government Tuesday announced it will release a Rs October – 1 November 2018.
75 commemorative coin on the occasion of 75th 188. The International Day to End Impunity for Crimes
anniversary of hoisting of Tricolour for the first time by against Journalist is a UN-recognized day observed
Netaji Subhash Chandra Bose at Port Blair. annually on 2nd November.
175. India's heaviest rocket, referred to as "Baahubali", 189. The National Green Tribunal has ordered a 4 member
successfully placed the sophisticated communications committee to conduct an inspection near the Polavaram
satellite GSAT-29 in the orbit today. India space agency Dam in Andhra Pradesh and submit a report after a plea
ISRO's Geosynchronous Satellite Launch Vehicle Mark alleged that muck was being dumped illegally at the site.
III (GSLV Mk-III-D2) - on its second developmental flight 190. Oscar-winning musician AR Rahman released his
- lifted off from Sriharikota, India's rocket port in Andhra biography, Notes of a Dream:The Authorized Biography
Pradesh on the coast of the Bay of Bengal. of AR Rahman, in Mumbai.Written by author Krishna
176. Delhi Police Commissioner Amulya Patnaik launched Trilok.
the force's e-learning portal NIPUN and training division 191. General VK Singh Minister of State for External Affairs,
website on November 14. Through the website, Delhi attended the 18th Indian Ocean Rim Association (IORA)
Police will be using advanced technology in imparting Council of Ministers Meeting in Durban,South Africa.
in-service training to its personnel 192. Vice Admiral MP Awati (retd.), who commanded a naval
177. Denmark said Thursday it would create an international unit of the Eastern Fleet in the 1971 India Pakistan War
research center focused on the fight against resistance to and whose actions led to the destruction of an enemy
antibiotics, a scourge affecting almost half a million submarine, passed away.
people worldwide. 193. South Korean first lady Kim Jung Sook and Chief
178. Paul Butler, Ph.D., and his colleagues at the Carnegie Minister Yogi Adityanath inaugurated the Queen Huh
Institution for Science found orbiting Barnard’s Star (the Memorial at Queen Huh Park in Ayodhya.
nearest singlestar to Earth, at six light years away) is 194. Uttar Pradesh Faizabad district will be known as
frozen solid — in other words, it’s nothabitable. Ayodhya after the temple town that is home to the Ram
179. The 62nd National Shooting Championship temple, said Chief Minister Yogi Adityanath.
Competitions (NSCC) 2018 in Small Bore Rifle and 195. The first Russia India Strategic Dialogue will take place
Pistol events (including Para events) will be organized in the Russian city of St Petersburg on November 25 and
by the National Rifle Association of India at 26.
Vattiyoorkkavu Shooting Range, 196. RBI has initiated steps to set up a wide-based digital
Thiruvananthapuram, Kerala from 15th November to Public Credit Registry(PCR) to capture details of all
7th December 2018 borrowers, including wilful defaulters and also the
180. Prime Minister, Narendra Modi, launched the pending legal suits in order to check financial
Government’s Support and Outreach Initiative for Micro, delinquencies.
Small and Medium Enterprises (MSME) at Vigyan 197. Uzbek businessman Gafur Rakhimov was elected president
Bhawan in New Delhi. The initiative will provide Small of theInternational Boxing Association (AIBA).
and Medium Enterprises easier access to credit, markets, 198. Veteran journalist and the Hindu Group of Publications
improved cash flows and online loans up to Rs 1 crore in Chairman N Ram has been chosen for the prestigious Raja
just 59 minutes. Ram Mohan Roy award, presented by the Press Council Of
181. The Maharashtra Cabinet on cleared the decks for the India, for his outstanding contribution towards journalism.
Navy’s longest serving aircraft carrier,INS Viraat, now 199. The World Tsunami Awareness Day is observed every year
across the world on 5th November to spread awareness
retired, to be converted into India’s first ever moored
among people across the world about Tsunami.
maritime museum-cum-marine adventure centre.
200. Railways UTS Mobile application for buying unreserved
182. Indian Air Force is organising a two day seminar on tickets online is available nationwide.
“Emerging Trends in aviation Engineering and

©LegalEdge Tutorials Page 59 of 60


Replication or other unauthorized use of this material is prohibited by the copyright laws of India
Page 60 of 60

Note:

©LegalEdge Tutorials
Replication or other unauthorized use of this material is prohibited by the copyright laws of India

Das könnte Ihnen auch gefallen